Загадки на логику по математике с ответами: Математические задачи — Логика и рассуждения

Содержание

Загадки Математические — Коллекция Загадок

Загадка № 4135

Дата: 29.06.2011, 00:01

В 9-этажном доме есть лифт. На первом этаже живет 2 человека, на втором — 4 человека, на третьем — 8 человек, на четвертом — 16, на пятом — 32 и так далее. Какая кнопка в лифте этого дома нажимается чаще других?

Ответ:Кнопка первого этажа.


Загадка № 4110

Дата: 19.06.2011, 20:45

Мельник пошел на мельницу и увидел в каждом углу по 3 кошки. Сколько ног на мельнице?

Ответ: 3*4*4+2=50


Загадка № 4108

Дата: 16.06.2011, 18:08

Два мальчика играли в шашки 2 часа. Сколько времени играл каждый мальчик?

Ответ: 2 часа


Загадка № 4102

Дата: 16.06.2011, 18:04

Термометр показывает плюс 15 градусов. Сколько градусов покажут два таких термометра?

Ответ: 15 градусов


Загадка № 4101

Дата: 16.06.2011, 17:59

Саша тратит на дорогу в школу 10 минут. Сколько времени он потратит, если пойдет вместе с другом?

Ответ: 10 минут


Загадка № 3926

Дата: 09.02.2011, 22:55

Кузнец подковал тройку лошадей. Сколько подков пришлось ему сделать?

Ответ: 12 подков


Загадка № 3925

Дата: 09.

02.2011, 22:54

Около столовой, где обедали лыжники, пришедшие из похода, стояли 20 лыж, а в снег было воткнуто 20 палок. Сколько лыжников ходило в поход?

Ответ: 10 лыжников


Загадка № 3924

Дата: 09.02.2011, 22:54

Сестре 4 года, брату 6 лет. Сколько лет будет брату, когда сестре исполнится 6 лет?

Ответ: 8 лет


Загадка № 3923

Дата: 09.02.2011, 22:54

Бабушка вязала внукам шарфы и варежки. Всего она связала 3 шарфа и 6 варежек. Сколько внуков у бабушки?

Ответ: 3 внука


Загадка № 3922

Дата: 09. 02.2011, 22:54

Дед, баба, внучка, Жучка, кошка и мышка тянули-тянули репку и, наконец, вытянули. Сколько глаз смотрело на репку?

Ответ: 12 глаз


Загадка № 3921

Дата: 09.02.2011, 22:54

Один профессор, который любил пить чай с сахаром, однажды, собираясь положить в чай ложку сахара, заметил, что ложка мокрая и к ней налип сахар. Экономка профессора, женщина строгая, заметила, что профессор рискует положить себе сахара больше, чем рассчитывал, т.к. сахар прилип к ложке. На что профессор гордо ответил, что экономка ничего не смыслит в математике и что он положит-таки ровно столько ложек сахара, сколько собирался. И он сделал это. Как?

Ответ: если бы песок прилипал только к наружной поверхности ложки, то задачки бы не было — насыпай, не размешивая, сколько надо и все.

.. Если предположить, что песок налипает внутри и снаружи одинаковой массой (х гр), то: если число ложек четное, то все просто — одну высыпаем (ложка-х), одну погружаем (ложка+х). Если число ложек нечетное, то последнюю можно аккуратно погрузить дно (х) и потом высыпать (ложка-х). Вопрос — как потом размешивать? Например, ручкой ложки


Загадка № 3920

Дата: 09.02.2011, 22:54

Мама старше сына на 21 год. Через 6 лет мама будет старше сына в 5 раз. Где папа?

Ответ: на маме (если X — возраст сына а Y — возраст мамы, то решая систему уравнений Х+21= Y и Y+6=5(X+6), получим: X=-3/4 года = -9 месяцев)


Загадка № 3919

Дата: 09.02.2011, 22:54

Ученый вывел новый сорт амеб.
Каждую минуту амеба делится пополам.
Профессор кидает в пробирку одну амебу, за час пробирка наполняется полностью.
За какое время пробирка наполнится, если туда кинуть не одну, а две амебы изначально?

Ответ: 59 минут


Загадка № 3918

Дата: 09.02.2011, 22:54

Hа затонувшей каравелле XIV века были найдены шесть мешков с золотыми монетами. В первых четырех мешках оказалось по 60, 30, 20 и 15 золотых монет. Когда подсчитали монеты в оставшихся двух, кто-то заметил, что число монет в мешках составляет некую последовательность. Приняв это к сведению, смогли бы вы сказать, сколько монет в пятом и шестом мешках?

Ответ: 60*1/2=30; 30*2/3=20; 20*3/4=15. Значит, 15*4/5=12 и 12*5/6=10


Загадка № 3917

Дата: 09. 02.2011, 22:53

Летело стадо гусей, увидел их мужик и говорит:
— Вас, поди, сто?
Они отвечают ему:
— Кабы нас столько, да ещё столько, да пол-столько, да четверть столька, да ты бы с нами – было бы сто.
Много ли гусей?

Ответ: 36


Загадка № 3916

Дата: 09.02.2011, 22:53

Сколько монет каждого из достоинств (25 центов, 50 центов и 1 доллар) могут составлять в сумме 700 долларов? Причем треть монет является четверть долларовыми, треть пол долларами и треть долларами?

Ответ: по 400 монет каждого достоинства


Загадка № 3915

Дата: 09.02.2011, 22:53

Вдоль улицы стоят 100 домов. Мастера попросили изготовить номера для всех домов от 1 до 100. Чтобы выполнить заказ, он должен запастись цифрами. Не пользуясь карандашом и бумагой, подсчитайте в уме, сколько девяток потребуется мастеру?

Ответ: двадцать


Загадка № 3914

Дата: 09.02.2011, 22:53

Два товарных поезда, оба длинной по 250 м,идут навстречу друг другу с одинаковой скоростью 45 км/ч. Сколько секунд пройдет после того, как встретились машинисты, до того как встретятся кондуктора последних вагонов.

Ответ: В момент встречи машинистов расстояние между кондукторами будет 500м. Т.к. каждый поезд идёт со скоростью 45км/ч относительно земли, то их скорость друг относительно друга 45+45=90км/ч или 25м/с. Искомое время равно 500:25=20 секунд.


Загадка № 3913

Дата: 09. 02.2011, 22:53

Один кирпич весит 1 килограмм и еще полкирпича.
Сколько весит один кирпич?

Ответ: 2 килограмма


Задачи на логику по математике

На одной далёкой планете есть только один аэропорт, находящийся на Севеном полюсе.
В распоряжении аэропорта имеются 3 самолёта и неограниченное количество горючего.
Бака самолёта хватает ровно до Южного полюса.
Самолёты имеют возможность дозаправляться (перекачивать горючее из одного в другой), находясь в полёте.
Как самолёт может облететь вокруг планеты так, чтобы все самолёты вернулись в аэропорт?

Решение:
Мысленно разделите весь путь от полюса до полюса на три части (от Северного полюса до Южного полюса 3 части. И от Южного полюса до Северного полюса 3 части). Стратегия состоит в следующем.
1. Два смолета летят до первой трети пути, один из них дозаправляет второй и поворачивает назад, а второй продолжает лететь до отметки 2/3 пути.
2. Два самолета вновь вылетают из аэропорта до первой третьей пути. Один из них дозаправляет второй и поворачивает назад, а второй продолжает путь до отметки 2/3 пути.
3. На отметке 2/3 пути теперь два самолета, бензобаки каждого заполнены на 2/3. Один из них дозаправляет второй и отправляется назад до отметки в 1/3 пути, где его встречает и дозаправляет 1/3 своего топлива третий самолет из аэропорта, оба летят обратно в аэропорт. В это время самолет, бывший на отметке 2/3 пути, с полным баком летит дальше, насколько это возможно (он пролетит над Южным полюсом, обогнет планету и долетит до отметки последней 1/3 пути до аэропорта).
4. Остальные действия очевидны. Из аэропорта вылетает еще один самолет (на этот раз в противоположном направлении, то есть навстречу обогнувшему планету самолету), чтобы поделиться 1/3 своего топлива с основным самолетом и вместе с ним вернуться в аэропорт.
———————
Волшебный пояс, исполняющий желания хозяина, уменьшается в два раза в длину и в 3 раза в ширину после каждого исполненного желания.
После исполнения трёх желаний площадь лицевой стороны стала 4 см2.
Какова была изначальная длина ремня, если его изначальная ширина была 9 см?

Ответ:
Изначальная длина пояса – 96см.
———————
У всех жителей города Болдвил разное количество волос на голове.
Нет ни одного жителя, у которого было бы точно 518 волос на голове.
Население города превышает число волос на голове любого из жителей Болдвил.
Каково максимально возможное население города Болдвил?

Решение:
Максимальное количество жителей города составляет 518 человек.
Причем один из жителей обязательно должен быть лысым, иначе в городе не будет ни одного человека.
———————
Кирпич весит 1 килограмм плюс половину собственного веса.
Сколько весит кирпич?

Решение:
Вам поможет обычное уравнение: 1 кирпич = 1кг + ½ кирпича, то есть x = 1 + x/2, откуда x — x/2 = 1,
откуда 2x — x = 2, откуда x = 2, то есть кирпич весит 2кг.
———————
Два поезда, находящиеся на расстоянии 200 км, движутся навстречу друг другу со скоростью 50 км/ч каждый. Муха берёт старт с одного из поездов и летит по направлению к другому со скоростью 75 км/ч. Долетев до другого поезда, муха разворачивается и летит назад к первому.
Так она летает туда и обратно, пока два поезда не сталкиваются, и насекомое погибает.
Какое расстояние успела пролететь муха?

Решение:
Расстояние, которое сможет пролететь муха за 2 часа (именно через два часа поезда столкнутся) с постоянной скоростью 75км/ч.
Она пролетит 150км.
———————
Из Бостона в Нью-Йорк выходит товарный поезд, двигаясь со скоростью 60 км/ч.
Через 30 минут навстречу ему из Нью-Йорка в Бостон выходит пассажирский поезд, двигающийся со скоростью 80 км/ч.
Который из поездов будет ближе к Нью-Йорку в момент встречи?

Решение:
Когда поезда встретятся, они оба будут приблизительно на одном и том же расстоянии от Нью-Йорка.
Поезд, выехавший из Нью-Йорка, будет ближе к Нью-Йорку примерно на расстояние, равное длине одного поезда, потому что поезда движутся во встречном направлении. Ну это если под словом «встретятся» Вы подразумеваете именно «встретятся», а не «пересекутся в тот самый момент, когда один из поездов поравняется всеми своими вагонами с вагонами второго поезда».
———————
Половину пути до города, находящегося на расстоянии 60 км, я проехал со средней скоростью 30 км/ч.
С какой скоростью я должен ехать остаток пути,
чтобы общая средняя скорость всего путешествия была бы 60 км/ч?

Решение:
В данном случае невозможно найти желаемую среднюю скорость. Простая арифметика покажет, что даже при скорости 90км/ч, или любой другой скорости, результат не будет удовлетворять условиям задачи.
———————
Мало что известно о жизни одного греческого математика из Александрии, которого называют родоначальником алгебры. Предполагается, что он жил в 3-м веке нашей эры. По рассказам, на его надгробии была высечена следующая эпитафия:
«Детство Диофанта 1/6 жизни заняло; 1/12 жизни Диофант бороду растил; ещё 1/7 жизни Диофанта прошла до того, как он женился. Через 5 лет после свадьбы у Диофанта родился сын, который прожил только половину лет, что прожил его отец. А через 4 года после смерти сына умер Диофант.»
Сколько лет прожил Диофант?

Решение:
Следующее уравнение отображает все периоды жизни Диофанта:
1/6x + 1/12x + 1/7x + 5 + 1/2x + 4 = x ;
Диофант прожил 84 года (x = 84).
———————
Сто мер зерна надо поделить между пятью работниками так, чтобы второй получил настолько больше, чем первый, насколько третий больше, чем второй, и на столько же, насколько четвёртый больше, чем третий, и на столько же, насколько пятый больше, чем четвёртый. Сколько мер зерна должен получить каждый, если первый и второй работники вместе получат зерна в семь раз меньше, чем остальные три работника?

Решение:
Для решения задачи составим два равенства. 5w + 10d = 100; 7*(2w + d) = 3w + 9d, где w – количество зерна для первого работника, d – разница в количестве зерна между двумя (следующими по порядку) работниками. Ответ: первому работнику 10/6 мер зерна, второму работнику 65/6 мер зерна, третьему работнику 120/6 (20) мер зерна, четвертому работнику 175/6 мер зерна, пятому работнику 230/6 мер зерна.
———————
Через два часа до полуночи останется в два раза меньше, чем оставалось бы через час.
Который сейчас час?

Ответ:
21:00
———————
В полдень часовая, минутная и секундная стрелки часов совпадают в одной точке циферблата.
Чуть больше, чем через час и пять минут, часовая и минутная стрелки совпадут снова.
Найдите с точностью до миллисекунды время, когда они совпадут.
Какой угол с ними будет в это время составлять секундная стрелка?

Решение:
Данная ситуация (когда часовая и минутная стрелки совпадают) повторяется 11 раз каждые 12 часов. Нетрудно догадаться, что отметка 1/11 окружности циферблата находится на моменте времени 1:05:27,273, то есть секундная стрелка будет стоять на отметке 27,273 сек.
Угол между часовой и секундной стрелкой в таком случае составит 131 градус.
———————
К бассейну подходят четыре трубы, по которым через краны можно контролировать скорость заполнения бассейна. Открыв первый кран, можно заполнить бассейн за 2 дня, второй – за 3 дня, третий – за 4 дня и четвёртый – за 6 часов.
Сколько понадобится времени, чтобы наполнить бассейн, открыв все четыре крана одновременно?

Решение:
Поскольку в сутках 24 часа, первый кран за час наполнит 1/48 бассейна, второй кран – 1/72, третий кран — 1/96, а четвертый наполнит бассейн на 1/6. Отсюда получаем: (6 + 4 + 3 + 48) / 288 = 61/288. Бассейн наполнится через 288/61 часов, то есть через 4 ч, 43 мин и примерно 17 сек.
———————
Военный автомобиль с важным посланием должен пересечь пустыню.
Однако полного бензобака хватает только на половину пути.
В распоряжении военной базы имеется несколько таких автомобилей, и бензин можно перекачивать из одного бака в другой.
Никакими канистрами и тросами они воспользоваться не могут.
Как доставить сообщение, не бросая ни одного автомобиля в пустыне?

Решение:
Всего понадобится 4 машины, включая ту, в которой находится ценное послание (та, что доедет до середины пустыни). Чтобы она пересекла пустыню и достигла место назначения, надо будет на середине пути заново заполнить бензобак под горлышко. Путь от военной базы (где машины и бензин) до середины пустыни можно условно поделить на три части. Каждая из трех вспомогательных машин короткими «перебежками» между условными отметками и базой сможет при каждой поездке сливать треть бензобака в другую вспомогательную машину, находящуюся ближе к главной машине.
За несколько поездок туда-сюда методом эстафеты вспомогательные машины в конечном счёте смогут полностью заправить главную машину, чтобы та смогла продолжить свой путь через вторую половину пустыни.
———————
Антрополог, изучавший племя в отдалённом уголке джунглей Амазонки, обнаружил странный обычай. Когда муж узнавал, что его жена изменяет, он должен был публично её казнить в полночь того же дня. Про любую женщину, изменяющую мужу, всегда знали все жители племени, кроме её мужа. Но мужу никто никогда не рассказывал об изменах его жены, потому что это противоречило кодексу чести. Тот же кодекс чести не позволял жёнам известить ту жену, чей муж был ей неверен. В противном случае она в тот же вечер пристрелила бы своего мужа. В день своего отъезда антрополог созвал всех представителей племени и объявил: «Я знаю, что в этом племени есть неверные жёны.»
И на девятый день все неверные мужья были казнены.
Сколько было неверных мужей?

Решение:
Если принять количество неверных мужей за число «n», то количество неверных мужей, известных каждой жене неверного мужа, составляет «n — 1» (потому что всем обо всём точно известно – только о верности собственного мужа приходится догадываться). Теперь построим следующую логическую цепочку.
Предположим, что количество неверных мужей равно единице. Тогда всем, кроме одной, жёнам известно, что среди жителей есть один неверный муж, в то время как жена этого неверного мужа уверена, что все мужья верны своим жёнам. Как только она услышит, что среди жителей есть как минимум один неверный муж, она тут же поймёт, что может быть только её муж, поэтому в тот же вечер она его незадумываясь застрелит.
А теперь представьте, что среди жителей есть два неверных мужа. Каждая жена таких неверных мужей уверена, что среди жителей есть только один неверный муж, поэтому ждёт, пока какая-нибудь из жён пристрелит своего мужа. Но в этот вечер никто никого не застрелил, а это может означать только одно: её собственный муж ей тоже неверен и является вторым неверным мужем в племени. Первая жена первого неверного мужа приходит к точно таким же выводам (она также ждала, что кто-то из жён застрелит своего мужа). Таким образом, обе оскорбленные жены в первый же вечер понимают, что их мужья им изменяют,
и на следующий вечер (второго дня) пристреливают обоих мужей.
Следуя этой логике, нетрудно догадаться, что количество неверных мужей «n» будут пристрелены в «n» — нный вечер.

Дальше:    логические задачи по математике с ответами >>>

Логические загадки и задачи на английском языке

Предлагаем вам интересные логические загадки на задачки логику на английском языке. Английские задачи на логику не только полезны для развития логического мышления, но и помогут обогатить словарный запас.

Логические загадки на английском

Предлагаем вам несколько блоков логических загадок, рассчитанных на разный уровень владения английским языком.

БЛОК 1. НОВИЧКАМ.

Для того, чтобы решить следующие логические загадки на английском, вам понадобится лишь смекалка, особых знаний английского они не требуют, загадки построены на базовой лексике.

  1. What is the largest ant in the world?
  2. David’s father has 3 sons: Adam, Bob and …. who?
  3. What appears once in every minute, twice in every moment, but not once in thousand years?
  4. What part of London is in Brazil?
  5. What is the beginning of everything, also the end of life?

БЛОК 2. СРЕДНИЙ УРОВЕНЬ.

Логические загадки второго блока потребуют от вас более глубокого знания лексики.

  1. What starts with E, ends with E and only has one letter?
  2. What start with a «t» ,end with a «t» and is full of «t»?
  3. What runs but never walks?

БЛОК 3. СЛОЖНЫЙ УРОВЕНЬ.

Сложный или нет, решать вам, но для того, чтобы успешно справится с логическими загадками данного уровня вам потребуется неплохое знание устойчивых выражений в английском, так как загадки в той или иной степени построены на игре слов.

  1. Which is faster, hot or cold?
  2. What kind of dog never bites?
  3. Why did I throw the butter out of the window?
  4. What is that you ought to keep after you give it to somebody else?
  5. When you say this word, it disappears. What is it?

Логические задачи на английском

Задача 1. Who’s who?

In Red Street, there live five people with five different jobs. Their names are Nick, Simon, Gregory, Mike and Nelly.

Their jobs are a vet, entrepreneur, doctor, writer and lawyer.

Their annual incomes are 3 million, 4 million, 5 million, 10 million, 20 million.

Their ages are 25, 35, 40, 50, 60.

They all have between 0-3 kids.

Read the facts about the people and complete the chart below:

Name

Age

Kids

Job

Income

Nick

 

 

lawyer

 

Simon

60

 

 

 

Gregory

40

3

vet

 

Mike

 

 

 

 

Nelly

 

 

 

3 million

 

  • Nick earns half as much money as Simon.
  • The entrepreneur earns the most.
  • Mike has as many children as Nick.
  • The oldest earns the second highest salary.
  • Nick is twice as old as Nelly.
  • The doctor has no kids.
  • Mike is younger than Gregory.
  • Nick has twice as many kids as Simon.
  • The person who has one kid is a writer.

Логическая задача 2. Birthday  Party!

Tim is 5 today.  He has invited five friends to his birthday party.  Can you tell who each child is? 

  • Kate is sitting next to Tim.
  • Margareth is between two boys.
  • Marik is opposite Lora.
  • Dan is sitting beside Marik.

Логическая задачка 3. Friends.

Fill in the chart according to the info below.

  1. Alex, whose birthday is in August is in the middle and he’s 8 years old
  2. Kate is 6 years old and she is the first, her birthday is 2 months before Alex’s
  3. On the right is Sam and she is 10 years old. Her birthday is 2 months after Alex’s
  4. Sophie’s birthday is in July and she is 7 years old,
  5. Jack is between Sam and Alex he is 9 years old, and his birthday is 3 months after Kate’s

Name

 

 

 

 

 

Birthday

 

 

 

 

 

Age

 

 

 

 

 

 

ОТВЕТЫ:

Блок 1.

  1. Elephant
  2. David
  3. Letter M
  4. Letter L
  5. Letter E

Блок 2

  1. envelope
  2. teapot
  3. Nose, river, etc.

Блок 3.

  1. Hot, you can easily catch cold
  2. A hot dog
  3. Because I wanted to see the butterfly.
  4. A promise
  5. Silence

Who’s who?

Name

Age

Kids

Job

Income

Nick

50

2

lawyer

5

Simon

60

1

write

10

Gregory

40

3

vet

4

Mike

35

2

entrepreneur

20

Nelly

25

0

doctor

3

Birthday  Party!

  1. Lora 2. Kate 3. Tim 4. Margaret 5. Dan 6. Marik

Friends

Name

Kate

Sophie

Alex

Jack

Sam

Birthday

June

July

August

September

October

Age

6 years old

7 years old

8 years old

9 years old

10 years old

 

Понравилось? Сохраните на будущее и поделитесь с друзьями!

Загадки обманки для детей с ответами

Загадки обманки — умопомрачительно весёлые загадки с неожиданными ответами для детей и их родителей.

В чем суть загадок обманок?

Суть загадок обманок для детей в том, что ответ, который хочется дать в рифму стишка, не является правильным.

Как правильно разгадывать загадки обманки?

Загадки обманки с ответами заканчиваются словом в рифму, но ответ должен быть совершенно не таким. Прочитайте полностью стишок и, вы увидите, что «на автомате» как дети, так и взрослые дают неверный ответ. Нужно совсем немножечко подумать, чтобы догадаться, какое слово является разгадкой на самом деле.

ВНИМАНИЕ: Правильный ответ указан в скобках при нажатии на слово «ответ».

Белым снегом всё одето —
Значит, наступает …

лето (зима)

ОТВЕТ

Ночью каждое оконце
Слабо освещает …

солнце (луна)

ОТВЕТ

Друг зверей и друг детей
Добрый доктор …

Бармалей (Айболит)

ОТВЕТ

Кукарекает спросонок
Милый, добрый …

поросёнок (петух)

ОТВЕТ

Высокий, длинноногий,
Летать ему не лень —
На крыше из соломы
Устроился …

олень (аист)

ОТВЕТ

Лишь только свет дневной потух,
заухал в темноте …

петух (филин)

ОТВЕТ

Чик-чирик! Чик-чирик! —
Кто поднял веселый крик?
Эту птицу не пугай!
Расшумелся …

попугай (воробей)

ОТВЕТ

Под деревом четыре льва,
Один ушёл, осталось …

два (три)

ОТВЕТ

Нашёл пять ягодок в траве
И съел одну, осталось …

две (четыре)

ОТВЕТ

Мышь считает дырки в сыре:
Три плюс две — всего …

четыре (пять)

ОТВЕТ

Кто взлетит с цветка вот-вот?
Разноцветный …

бегемот (мотылёк)

ОТВЕТ

С пальмы вниз, на пальму снова
Ловко прыгает …

корова (обезьяна)

ОТВЕТ

Простой вопрос для малышей:
Кого боится кот?..

мышей (собак)

ОТВЕТ

Хвост веером, на голове корона.
Нет птицы краше, чем …

ворона (павлин)

ОТВЕТ

Мимо улья проходил
Косолапый …

крокодил (медведь)

ОТВЕТ

В чаще, голову задрав,
воет с голоду …

жираф (волк)

ОТВЕТ

Дочерей и сыновей
учит хрюкать …

соловей (кабан)

ОТВЕТ

Кто грызёт на ветке шишку?
Ну, конечно, это …

мишка (белка)

ОТВЕТ

Ква-ква-ква — какая песня!
Что быть может интересней,
Что быть может веселей?
А поет вам …

соловей (лягушка)

ОТВЕТ

Скорей на берег выбегай!
Плывет зубастый …

попугай (крокодил)

ОТВЕТ

Как? Неизвестно до сих пор:
секрет и есть секрет,
зверь этот, словно светофор,
свой изменяет цвет.
В зелёный, жёлтый… Напугай —
и покраснеет …

попугай (хамелеон)

ОТВЕТ

Cъела зайчонка и ловит второго
огненно-рыжая злая …

корова (лиса)

ОТВЕТ

Он пиявок добывал,
Карабасу продавал,
Весь пропах болотной тиной,
Его звали…

Буратино (Дуремар)

ОТВЕТ

Он гулял по лесу смело,
Но лиса героя съела.
На прощанье спел бедняжка.
Его звали …

Чебурашка (Колобок)

ОТВЕТ

Много дней он был в пути,
Чтоб жену свою найти,
А помог ему клубок,
Его звали …

Колобок (Иван-Царевич)

ОТВЕТ

И красива, и мила,
Только очень уж мала!
Стройная фигурочка,
А зовут …

Снегурочка (Дюймовочка)

ОТВЕТ

Я красивый, я летаю,
А весной от солнца таю.
Угадайте поскорей,
Кто же это? …

воробей (снег)

ОТВЕТ

Он большой шалун и комик,
У него на крыше домик.
Хвастунишка и зазнайка,
А зовут его …

Незнайка (Карлсон)

ОТВЕТ

Утром рано я встаю,
Молочком всех напою,
Травку я жую за речкой,
А зовусь я как? …

овечка (корова)

ОТВЕТ

Жил в бутылке сотни лет,
Наконец, увидел свет,
Бородою он оброс,
Этот добрый …

Дед Мороз (Старик Хоттабыч)

ОТВЕТ

Все меня боятся —
Я могу кусаться,
Я летаю и пищу —
Жертву я себе ищу,
Ночью мне уж не до игр,
Угадали, кто я? …

тигр (комар)

ОТВЕТ

Я лаю и кусаю,
Я дом ваш охраняю,
Всегда смотрю во все глаза,
А как зовут меня? …

коза (собака)

ОТВЕТ

С голубыми волосами
И огромными глазами,
Эта куколка — актриса,
А зовут ее …

Алиса (Мальвина)

ОТВЕТ

И капризна, и упряма,
В детский сад не хочет …

мама (дочка)

ОТВЕТ

На прививки и уколы
Мамы деток водят в …

школы (поликлиники)

ОТВЕТ

Загадки, викторины, головоломки: 10 загадочных диафильмов

Одна из прекрасных форм совместного досуга детей и взрослых, гарантирующая нескучный вечер в кругу семьи, – диафильмы. А диафильмы с загадками, викторинами и головоломками помогут провести время не только весело, но и с пользой!

 

Начнем с математики. Обратите внимание, что викторины рассчитаны на разный возраст.

 

Диафильм «Математическая викторина» 

Данное пособие может быть использовано для занятий по развитию элементарных математических представлений в работе с детьми старшей группы детского сада или в семье.

Прямо наш случай – используйте в семье, не забывайте хвалить и поощрять детей за правильные ответы на вопросы викторины.

 

Диафильм «Занимательная математика. 1 класс»

Как следует из названия, этот диафильм – для ребят, которые уже ходят в первый класс школы. В диафильме даются задания на развитие внимания, наблюдательности, пространственных представлений, задачи по арифметике, логике, геометрии.

 

Диафильм «Занимательные задачи по математике. 3 класс»

 

А еще можно и историю метро вспомнить. На картинке схема московского метро 1977 года.

Диафильм можно использовать на октябрьских и пионерских сборах и при проведении конкурсов.

Вот и будем использовать для проведения семейных мини-конкурсов! Хотя при современном уровне развития конференц-связи можно и настоящий конкурс организовать.

 

Диафильм «Рассказы о математике. Алгебра»

Этот диафильм – для ребят постарше, учеников седьмого класса. Посвящен алгебре.

С тех пор Оля, как только хотела пожаловаться на математику, приходила к Соседу, который знал всё.

Мы к соседу не пойдем, пообщаемся с ним через экран, это не менее интересно.

 

Теперь переходим к географии. Будьте внимательны: часто ответы на географические вопросы надо давать с учетом исторических перемен – все диафильмы были сделаны в СССР, а сейчас многие правильные ответы уже вовсе не в «нашей стране». Но от этого еще интереснее!

 

Диафильм «Путешествие по родной стране»

Это диафильм-игра, и вам понадобятся бумага и карандаш, так как в первой половине диафильма идут вопросы, а во второй –  ответы. Именно в этом диафильме важно помнить, что теперь родная страна не включает союзные республики.

 

Диафильм «Угадай-ка!»

Диафильм построен по принципу вопрос-ответ: один кадр – вопрос, следующий – ответ, так что листайте аккуратно.

 

Диафильмы «Географические загадки»

Это универсальный диафильм, построенный по принципу вопрос-ответ. Загадки будут интересены детям и взрослым.

 

Не обошли занимательным подходом и русский язык:

 

Диафильм «Буратино — деревянный человечек» 

Этот диафильм сделали для уроков русского языка и развития речи в первом классе. Конкретно – для изучения темы «Слова, отвечающие на вопросы Какой? Какая? Какое?». Проверьте себя и детей, заодно вспомните сюжет сказки про Буратино.

 

Напоследок немного экзотики:

 

Диафильм «Астрономические загадки»

Хорошо ли вы знаете астрономию? Ведь в каждой квартире есть предметы, связанные с астрономией. Знаете, что это за предметы?

 

Диафильм «Викторина по гражданской обороне для учащихся общеобразовательных школ»

Этот диафильм-викторина – шанс блеснуть для пап, которые в школе изучали все, о чем идет речь в диафильме: поражающие факторы ядерного взрыва, типы противогазов и т.п. Неожиданный бонус – выкройка ватно-марлевой повязки.

 

Все диафильмы находятся в свободном доступе в Национальной электронной детской библиотеке. 

 

Текст: канал НЭДБ на Яндекс.Дзен.

 

Загадки, викторины, головоломки – интеллектуальный спорт для всей семьи! Будьте здоровы!

Математические загадки на логику для детей читать онлайн

Какие три числа при сложении и умножении дают один результат?

( 1 + 2 + 3 = 6, 1 * 2 * 3 = 6)
* * *

Больше часа, меньше минуты.

(секунда)
* * *

В парке 8 скамеек. Три покрасили.
Сколько скамеек стало в парке?

(восемь)
* * *

Какой знак нужно поставить между 6 и 7, чтобы результат оказался меньше 7 и больше 6?

(запятую)
* * *

В комнате 4 угла. В каждом углу сидела кошка, напротив каждой кошки — 3 кошки.
Сколько кошек находилось в комнате?

(4 кошки)
* * *

Как 2 литра молока поместить в литровую банку?

(получить творог)
* * *

Шли муж с женой, брат с сестрой да муж с шурином.
Сколько всего человек?

(три человека)
* * *

На столе лежало 4 яблока. Одно из них разрезали пополам и положили на стол.
Сколько яблок осталось на столе?

(4 яблока)
* * *

На столе лежит 100 листов бумаги.
За каждые 10 секунд можно посчитать 10 листов.
Сколько секунд понадобится, чтобы посчитать 80 листов?

(20)
* * *

На столе лежат линейка, карандаш, циркуль, резинка.
На листе бумаги нужно начертить окружность.
С чего начать?

(с листа бумаги)
* * *

В каком числе цифр столько, сколько букв в его названии?

(100 — сто, 1000000 — миллион)
* * *

Горело 7 свечей. 2 свечи погасили.
Сколько свечей осталось?
(7 свечей)
* * *

Сколько различных цифр надо использовать, чтобы написать число 100?

(две — 0 и 1)
* * *

В каком случае число 1322 меньше 622?

(года до нашей эры)
* * *

В каком слове 3 буквы л и три буквы п?

(параллелепипед)
* * *

Пара лошадей пробежала 40 км.
По сколько километров пробежала каждая лошадь?

(40 км)
* * *

Сколько земли содержит яма диаметром 3 метра и глубиной 3 метра?

(нисколько, ямы пустые)
* * *

Какими нотами можно измерить расстояние?

(ми-ля-ми)
* * *

В комнате было 12 цыплят, 3 кролика, 5 щенят, 2 кошки, 1 петух и 2 курицы.
В комнату вошёл хозяин с собакой.
Сколько в комнате стало ног?

(две, у животных нет ног)
* * *

Было два отца и два сына. И всего три яблока. Каждый съел по яблоку.
Как такое возможно?

(сын, отец, дедушка)
* * *

У меня нет веса, но я бываю лёгкой, бываю тяжёлой.
Кто я?

(музыка)
* * *

В люстре горело пять лампочек. Две из них погасли.
Сколько лампочек осталось в люстре?

(осталось 5)
* * *

Суммарный возраст отца и сына — 66 лет.
Возраст отца — это возраст сына, записанный справа налево.
Сколько лет каждому?

(51 и 15, 42 и 24, 60 и 06)
* * *

Электровоз идёт на запад со скоростью 90 км/ч.
Дует восточный ветер, скорость ветра 10 км/ч.
В каком направлении идёт дым?

(у электровоза нет дыма)
* * *

Пять пломбиров пять ребят

Ровно в пять минут съедят.

А за сколько смогут съесть

Шесть ребят пломбиры, если

И пломбиров тоже шесть?

(сколько б ни было пломбиров,

если столько же ребят,

то ребята все пломбиры

в те же пять минут съедят)
* * *

Сколько раз можно вычесть 6 из 30?

(только один, потом 6 будет вычитаться из 24)
* * *

Две матери, две дочери и бабушка с внучкой.
Сколько всех?

(трое: бабушка, мать, внучка)
* * *

Как число 666 увеличить в полтора раза, не производя над ним никаких арифметических действий?

(написать 666 и повернуть её вверх ногами)
* * *

Когда мы смотрим на цифру 2, а говорим 10?

(когда смотрим на часы)
* * *

Сколько будет 2+2 х 2=?

(6)
* * *

Что тяжелее: килограмм железа или килограмм пуха?

(вес одинаков)
* * *

Одно яйцо варится 3 минуты.
Сколько минут будут вариться 2 яйца?

(3 минуты)
* * *

Сколько полосок у зебры?

(две: черная, белая)
* * *

Сколько яиц можно съесть натощак?

(одно, остальные не натощак)
* * *

Грузовик ехал в деревню.
По дороге он встретил 4 легковые машины.
Сколько машин ехало в деревню?

(одна)
* * *

Простая логическая загадка, демонстрирующая нелогичность людей / Хабр


Питер Васон

В 60-х годах психолог Питер Васон придумал эксперимент-загадку, «Задача выбора Васона». Говорят, что это наиболее часто исследуемая задача в психологии принятия решений.

Васон отличался чувством юмора и необычным мышлением. Он исповедовал отношение к психологии принятия решений как к загадке, которую надо изучать как критически, так и с долей развлечения. Он говорил своим коллегам, что будет изучать их работы только после того, как проведёт свои эксперименты, чтобы не искажать свою точку зрения. Также он сказал, что экспериментаторы никогда не должны точно знать, зачем они проводят эксперимент. «Целью его экспериментов было не проверить гипотезу, а изучить сущность мышления»,- так написали его ученики в его некрологе в 2003 году. «Он всегда хотел продемонстрировать некий феномен, чтобы показать, что мышление не такое, каким его представляют психологи, включая его самого».

Одна из версий задачи звучит так – испытуемому (который был всегда один, ибо Васон избегал групповых тестов) предлагались четыре карты – с одной стороны у каждой было число, с другой – один из двух цветов. Допустим, вы – испытуемый. У первой и второй карт вы видите лицевую сторону с числами 5 и 8, у третьей и четвёртой – обратную сторону, у одной – голубую, у другой — зелёную.

Экспериментатор сообщает вам следующее утверждение: если у карты на лицевой стороне изображено чётное число, то её обратная сторона – голубая. Вопрос: какие карты необходимо перевернуть для проверки этого утверждения?

Прим. перев. Мне показалось, что среди программистов и IT-специалистов распределение правильных ответов должно быть несколько другим, нежели 10% на 90%. Поэтому я потрудился сделать опрос. Если вам не трудно, перед просмотром правильного ответа выберите вариант, который вам нравится, в опросе внизу страницы.

Видео – интерактивное, там можно выбрать один вариант и нажать на него, чтобы проверить себя.

Если вы ошиблись – не отчаивайтесь. Задача проста лишь на первый взгляд. По статистике, справляются с ней лишь 10% людей.

Правильный ответ: — «5» переворачивать не нужно, т.к. в утверждении ничего не сказано про карты с нечётными числами. Утверждение «у карт с чётными числами обратная сторона – голубая», не означает автоматически, например, что у карт с нечётными числами должна быть зелёная обратная сторона
— «8» перевернуть надо, чтобы проверить, действительно ли у неё обратная сторона – голубая
— голубую карту не надо переворачивать – какое бы число там не было изображено, это не нарушит утверждения
— зелёную карту нужно перевернуть – если там окажется чётное число, то утверждение будет опровергнуто

Итого, необходимо перевернуть карты «8» и зелёную.

Васон опубликовал в 1968 году работу, в которой назвал результаты экспериментов «тревожными», поскольку изначально предполагалось, что люди размышляют аналитически – но, как оказалось, на деле их рассуждения иррациональны. Васон задумался: была ли виной тому логическая структура правил игры, или людей сбивали с толку слова, формулирующие задачу?

В 1982 пара психологов, Ричард Григс и Джеймс Кокс попытались доказать, что сложность задачи проистекает из её формулировки. Они переформулировали задачу так: представьте, что вы полицейский, ищущий несовершеннолетних посетителей баров. Если человек пьёт пиво, его возраст должен быть больше 21 года. В этом случае правильно ответили на вопрос 75% испытуемых. Этот эффект назвали «эффект контекста» — влияние факторов окружающей среды на восприятие человеком стимула. То есть, сложность задачи зависит от её описания, хотя смысл её остаётся тем же. Но почему же слова оказывают такое влияние, если логическая структура не меняется?

В книжке от 2011 года «Мышление, быстрое и медленное», психолог Дэниел Кахнеман представил теорию «двойной обработки». Она утверждает, что разные слова заставляют вас использовать одну из двух систем распознавания. «Старая система», заложенная в человеке, интуитивная, или «система 1», работает быстрее, а «новая», «система 2» – медленнее. Столкнувшись с задачей, две системы начинают борьбу за выдачу правильного ответа. Если вы ошиблись с ответом – вините старую, интуитивную «систему 1», которая любит «срезать углы», работая со «склонностью к шаблонам». Люди склонны, отвечая на вопрос, выбирать те варианты, которые были упомянуты в самом вопросе. Поэтому чаще всего они выбирают карту «8» (правильный выбор) и голубую (неправильный), потому, что в вопросе есть слова «чётный» и «голубой». Но переворачивать голубую карту нет смысла, поскольку, что бы там ни было на другой стороне, это не будет противоречить утверждению.

Но отчего же люди склонны к нерациональным поступкам? Потому, что «это быстрее и кажется правильным», как написал один из психологов. Напротив, абстрактные рассуждения, которым занимается новая система – штука утомительная. Часто новая система, не осознавая этого, пропускает старую вперёд. Поэтому Кахнеман писал, что «одна из главных её характеристик – лень». Но в случае с пивом и совершеннолетием старая система справляется лучше, потому, что ей известны правила насчёт спиртных напитков и возраста, и она использует их, чтобы решить задачу.

Объяснение психологии методом «двойной обработки» получает подтверждение в других исследованиях. Например, если вы ответили правильно, скорее всего на вступительных экзаменах в институт вы наберёте больше баллов чем те, кто её не решил. Также была показана деградация «новой» системы по сравнению со «старой» с возрастом. Это объяснение делит наш разум на две части – новую систему принятия решений и старую, которая работает на автопилоте без особого контроля.

Если вас это не убедило – вы не одиноки. Эволюционные психологи сомневаются, что случайная эволюция так чётко разделила бы мышление на два разных процесса. С их точки зрения, людям легче решить задачку в форме, связанной с баром потому, что люди, как они говорят, эволюционировали в «социо-когнитивной нише». Им необходимо было адаптироваться, чтобы создавать и следовать правилам, чтобы функционировать, как «уникальный и конкурентный хищный организм».

Поэтому, с точки зрения эволюционного психолога, вариант с баром говорит о том, что человеку легко сказать, кто именно нарушает правила, принятые в социуме. В 1989 году Леда Космидес назвала эту возможность «модуль обнаружения жулика». С тех пор её коллеги использовали задачу выбора Васона для проверки социального фактора людей. В одном из исследований они установили, что родителям свойственно сильнее заботится о собственных детях. Родители хуже справлялись с заданиями, связанными с обработкой правил безопасности, если речь шла о чужих детях.

Сам Васон решил отстраниться от научных дебатов касательно его работы, и с удовольствием наблюдал рождение новых идей, связанных с ней. Он заинтересовался шахматами, написал книгу под названием (сюрприз) «Психология шахмат» и добился в ней звания гроссмейстера международного уровня. Как он позже писал про самого себя: «Возможно, меня всегда привлекала тема рассуждений и логики, потому, что большинство вещей в жизни – нелогичны».

30 лучших математических головоломок, которые должен попробовать каждый

Интервью Math Riddles Puzzles Комментариев нет

Математические головоломки или математические головоломки основаны на логике и немного математике. Это очень важно для развития логического ума. Человек, имеющий привычку решать математические головоломки, развивает логическое мышление и считается человеком с очень высоким IQ. Эти головоломки не зависят от возрастной группы, поскольку для этого требуется лишь немного математики, но высокая логика.

Математические головоломки в наши дни становятся интересом работодателей во время собеседований, что позволяет работодателям проверять IQ кандидата. Здесь мы перечислили 30 самых известных математических головоломок, которые должен решить каждый в любой возрастной группе. Когда вы разберетесь со всеми 30 головоломками, вы будете хорошо подготовлены к любым головоломкам на собеседовании.

Решайте 2 головоломки из этого списка ежедневно, и все готово! 🙂

Список из 30 математических головоломок с ответами, основанных на математике и нестандартной логике (не пропустите ни одного, попробуйте все) —

  1. Неисправный шар из 8 шаров Пазл
  2. Проблема с горящей веревкой
  3. Пазл Famous Handshake
  4. Пазл с 10 монетами с завязанными глазами
  5. Знаменитая головоломка с математическим уравнением
  6. Пазл Верблюд и банан
  7. Пазл лавочник и поддельная записка
  8. Соотношение мальчиков и девочек в стране, где люди хотят только мальчиков
  9. Пазл для измерения 5 литров
  10. Вероятностная головоломка Marbles
  11. Жадные пираты и раздача золотых монет Головоломка
  12. Пазл о королевских бутылках и бутылках вина
  13. Пазл переходя мост
  14. Пазл Случайные места для самолетов
  15. 2 яйца и 100 этаж Google Classic вопрос
  16. Отмерьте 9 минут из пазла 2 песочных часа
  17. Пазл «Убийство и выживание на мечах» (проблема Иосифа)
  18. Пазл неисправная коробка для шоколада
  19. Красный и синий шарики Пазл
  20. Проблема с золотым слитком
  21. Пазл на 100 дверей
  22. Пазл Луковицы в круге
  23. Пазл на путаницу сумм и произведений
  24. Наибольшее число с использованием четырех троек Головоломка
  25. Пазл, образующий 6-значное число
  26. Математическая головоломка умножения пальцев
  27. Задача о муравье и треугольнике
  28. Загадка инвестора с Уолл-стрит
  29. Головоломка банковского кассира
  30. Пазл куб Каландара

Готово? Ух ты!! теперь вы чемпион по математике

математических головоломок

математических головоломок

    Эти головоломки не требуют никаких математических знаний, только логические рассуждения.Проверь, насколько ты умный. Если вы не можете их решить, расслабьтесь. Почти все головоломки нам рассказали компьютерно-математический гений Влад Митлин. Приходите к нам снова: мы намерены размещать здесь новые головоломки и решения.

    Присылайте нам свои комментарии и новые головоломки по электронной почте: [email protected]. Наслаждаться!


1. Монеты
    Есть 12 монет. Один из них ложный; у него другой вес. Неизвестно, тяжелее или легче фальшивая монета, чем правильные монеты.Как найти фальшивую монету по трем весам на простых весах?
Решение: кликните сюда

2. Мостовой переход


Эта проблема была недавно опубликована в MAA на сайте: Переход через шаткий мост ночью при свете фонарика.
    Группа из четырех человек должна перейти мост. Темно, и они должны осветить путь фонариком. Одновременно мост могут пересечь не более двух человек, а у группы есть только один фонарик. Люди в группе переходят мост разное время:
      A nnie пересекает мост за 1 минуту,
      B ob пересекает мост за 2 минуты,
      Володя Митлин пересекает мост за 5 минут,
      D Ороти пересекает мост за 10 минут.
Как группе пройти через мост за 17 минут?

Решение:

кликните сюда
Чтобы увидеть анимированное решение, вам нужен браузер, поддерживающий JAVA.

3. Доставка яблок


Еще проблемы от Влада Митлина


Вечеринка!

На вечеринке группа людей. Покажите, что вы можете представить некоторые из их друг к другу так, чтобы после знакомства не более двух человек в группе будет одинаковое количество друзей (первоначальная конфигурация не работает, потому что у всех изначально 0 друзей).


Цифры

Покажите, что для любого натурального n хотя бы одно из двух чисел, n или n + 1, может быть представлены в следующем виде: к + S (к) для некоторого k, где S (k) — сумма всех цифр в k. Например, 21 = 15 + (5 + 1)


Проблема дзен

Буддийский монах получил задание от своего учителя: медитировать ровно 45 минут. У него нет часов; вместо этого ему дают две ароматические палочки, и он говорят, что каждая из этих палочек полностью сгорит за 1 час.В палочки не идентичны, и горят с разными, пока неизвестными скоростями (они сделаны вручную). Итак, у него есть эти два аромата и несколько совпадений: он устраивает медитацию ровно на 45 минут?


Счастливые билеты

В России садишься в автобус, берешь билет и иногда говоришь: Вау, а счастливое число! Автобусные билеты нумеруются шестизначными числами, а счастливый в билете сумма трех первых цифр равна сумме трех последних цифр. цифры. Когда мы учились в старших классах (ребята из математической школы No.N — 1


Расстояния

В прямоугольнике 6 точек со сторонами 3 и 4. Докажите, что расстояние между как минимум двумя из этих точек меньше квадрата корень 5.


король

Шахматный король помещается на шахматную доску 8×8. 2 + ab + ac + ad + bc + bd + cd не меньше 10.


Три планеты в галактике и обвал рынка

Галактика состоит из трех планет, каждая из которых движется по прямой линия с собственной постоянной скоростью. Если центры всех трех планет случайно лежат на прямой (какое-то затмение) жители каждая планета сходит с ума (они не могут видеть свои две соседние планеты сразу один раз), начните говорить о конце света, а фондовый рынок вылетает. Покажите, что таких обвалов рынка будет не более двух. каждая из этих планет.4 есть решения в простых числах? Найдите хотя бы один, если да, в противном случае приведите доказательство несуществования.


Последовательность

Последовательность натуральных чисел определяется по следующей формуле: A [n + 1] = a [n] + f (n) Где f (n) — произведение цифр в [n]. Есть ли [1] такая, что указанная выше последовательность не ограничена?


Интеллектуальная сила стаи драконов

Драконы должны встретиться для мозгового штурма в конференц-центре.В делегаты должны быть отобраны, чтобы обеспечить максимальную эффективность мозговой штурм. У дракона любое количество голов, и для любого N при необходимости доступно любое количество N-головных драконов. Проблема в что размер конференц-центра ограничен, поэтому не более 1000 головы могут поместиться в актовый зал. Интеллектуальная сила стая драконов — это произведение количества голов драконов в стае. Как должна выглядеть оптимальная стая (общее количество драконов, распределение голов)?


Истребитель вампиров

На поверхности планеты живет вампир, который может перемещаться скорость не больше u.Космический корабль истребителей вампиров приближается к планеты со скоростью v. Как только космический корабль видит вампира, он стреляет серебряной пулей — вампир мертв. Докажите, что если v / u> 10, убийца вампиров может выполнить свое миссии, даже вампир пытается спрятаться.


Проекторы

(2D) проектор освещает квадрант на плоскости. 4 проектора установить в 4 произвольных точках плоскости. Покажи, что их можно повернуть так что весь самолет будет освещен.(3D) Покажите, что все пространство может быть освещен 8 прожекторами, каждый из которых освещает октант, однако точки расположения есть.


Кампания бдительности в Солт-Лейк-Сити.

Солт-Лейк-Сити выглядит как прямоугольник, пересеченный улицами M , идущими от С севера на юг и с улицами N и , идущими с востока на запад. Город часто посещают туристы, которые предполагают бегать в автобусах. Губернатор штата Юта хочет следить за всеми перемещениями автобусов.Он планирует выставить на некоторых перекрестках полицейских, чтобы они наблюдали за движением автобусов. улицы, видимые с этих перекрестков. Какое минимальное количество полицейских необходимо для наблюдения за автобусом?


Блондинки (загадка из Oldaque P. de Freitas)

Две блондинки сидят в уличном кафе и разговаривают о детях. Один говорит, что у нее три дочери. Произведение их возраста равно 36 а сумма возрастов совпадает с номером дома через улицу.Вторая блондинка отвечает, что этой информации недостаточно, чтобы выяснить возраст каждого ребенка. Первый соглашается и добавляет, что у старшей дочери красивые голубые глаза. Затем второй решает загадку. Вы тоже можете это решить!


Это от Гжегожа Дзержановского.

Есть 12 зубочисток. Найдите многоугольники экстремального поля, используя все зубочистки. При построении этих полигонов соблюдайте правила: — зубочистки не сломаешь, — длина каждого ребра 1,2,3,… зубочистки, — края многоугольника не могут пересекать друг друга.



Помогите ребенку развить математические навыки с помощью логических головоломок. — IMACS

Пусть математика будет увлекательной, и ваши дети будут продолжать заниматься математикой. Привет, логические головоломки!

Логические головоломки для детей — отличный способ пробудить интерес вашего ребенка к математике, поскольку логика и математика идут рука об руку. Каждый раз, когда дети увлекаются сложными головоломками, они развивают навыки, которые в конечном итоге будут использовать, чтобы сдать экзамен SAT, привести команду к победе на Первом соревновании по робототехнике и даже разделить счет в ресторане со своими будущими соседями по комнате.

Регулярное решение головоломок приносит больше пользы. Исследования показывают, что игра в интеллектуальные игры развивает навыки аналитического мышления, давая учащимся возможность неожиданно использовать стратегии решения проблем. Например, в English 101, когда вашему ученику нужно связать тезис с заключением, он будет вызывать свой набор навыков логического мышления. В робототехнике, если созданный ими робот для изготовления бутербродов кладет хлеб на сыр (вместо сыра на хлеб), аналитическое мышление позволит им отладить его и есть стильно.

Будь то задача ребуса, проблема нестандартного мышления или умопомрачительная головоломка судоку, каждая загадка делает вашего ребенка (и вас) лучшим писателем, мыслителем, стратегом и математиком.

Что такое логика и почему она необходима для критического мышления?

Слово логика происходит от древнегреческого слова logos, первоначально означающего «слово» или «то, что говорится». Однако со временем этот термин стал означать «мысль» или «разум».«Логическая головоломка — это в буквальном смысле мысленная головоломка! Этимология объясняет, почему существует так много разновидностей логических головоломок для детей — от математических головоломок, построенных исключительно на отношениях между числами, таких как «24», до сложных загадок с игрой слов. Проверьте это:

Что становится влажнее и влажнее, чем больше сохнет?

Кто может бегать, но никогда не ходит, имеет рот, но никогда не разговаривает, имеет голову, но никогда не плачет, имеет кровать, но никогда не спит?

Я невесом, но вы меня видите. Положите меня в ведро, и я сделаю его легче.Что я?

В тупике? Вы найдете ответы внизу статьи.

Вау! Существует так много типов логических головоломок .

Решение проблем включает в себя широкий спектр когнитивных инструментов, таких как индуктивная логика, дедуктивные рассуждения, аналогии, игра слов, упорядочивание и даже иногда просто очень, очень внимательное слушание.

Вот классический пример головоломки, которая полностью полагается на то, что ученые называют метакогнитивным слушанием, что является просто причудливым способом сказать: «Обращайте пристальное внимание на то, что задает вопрос.”

Фермер в Калифорнии владеет красивой грушей. Он поставляет фрукты в ближайший продуктовый магазин. Владелец магазина позвонил фермеру, чтобы узнать, сколько фруктов он может купить. Фермер знает, что у главного ствола 24 ветви. В каждой ветке ровно 12 ветвей, а в каждой — ровно шесть веток. Поскольку каждая веточка приносит по одному фрукту, сколько слив фермер сможет доставить?

Подождите? Какие! Вы сделали связку умножения? Вот подсказка: в этом нет необходимости.Почему? Что ж, приятно знать так много о прекрасном грушевом дереве фермера, но он разносит сливы!

Как у покупателя в магазине мороженого есть варианты, так и у поклонника логической головоломки. Как и в случае с мороженым, проблемы бывают разных вкусов, и у людей обычно есть свои любимые. Давайте изучим несколько и посмотрим, что вам больше всего нравится.

Классические математические задачи рассуждений.

Учебная программа IMACS направлена ​​на развитие у учащихся навыков математического мышления, поэтому неудивительно, что IMACS создал сотни уникальных логических задач.Попробуйте это из библиотеки IMACS:

Был морозный зимний вечер, когда сестры Аврора, Элизабет, Минни и Ророна сидели у камина и пили горячее какао. Когда они играли и пили из своих чашек, одна из сестер случайно пролила какао на ковер. Их мать вскоре проверила своих дочерей и спросила, взяв чашку: «Кто пролил какао?» Правду сказала только одна из сестер.

Элизабет сказала: «Чаша принадлежит Ророне.

Ророна сказал: «Чаша принадлежит сестре, в имени которой есть буквы, составляющие слово« рев »».

Аврора сказала: «Чаша принадлежит сестре с двумя буквами« о »в ее имени. . »

Минни просто смотрела на какао на полу.

Кто виновен в пролитии какао?

Этот тип логической задачи может оказаться чрезвычайно сложной задачей, прежде чем вы разовьете свои навыки рассуждения.Но во многих отношениях это небольшое препятствие для мышления, которое в малом масштабе имитирует сложности, с которыми изобретатели, инженеры и технологи сталкиваются в реальной жизни: оно требует понимания взаимосвязанных отношений и способности применять дедуктивное мышление. Любая сложная головоломка улучшит умственную гибкость человека, но логическое рассуждение является наиболее важным для развития, поскольку оно лежит в основе многих других важных стратегий решения проблем.

К счастью, логическое рассуждение является усвоенным, а не врожденным.Программа IMACS специально разработана для развития навыков мышления, необходимых для решения более сложных задач. Фактически, IMACS даже предлагает каждое лето веселые и увлекательные уроки, посвященные логическим головоломкам для детей с первого по восьмой класс.

Ищете ответ на проблему пролитого какао? Вы можете найти его (и другие) в конце статьи. Пожалуйста, не расстраивайтесь, если эти проблемы сложны! Имейте в виду, что математические рассуждения имеют много общего с изучением языка:

  • лучше и легче всего овладеть им в молодом возрасте;
  • Вы должны потратить время, чтобы научиться бегло говорить;
  • Как только вы начнете свободно говорить, навык станет настолько автоматическим, что кажется, будто вы всегда знали, как это делать!

Хотите попробовать другой? Большой! Попробуйте:

Лори, Мэтт и Нэнси заказали бутерброд, гарнир и напиток в ресторане быстрого питания.Когда их заказы были доставлены: Лори получила бутерброд с рыбой, картофель фри и воду. Мэтт получил бутерброд с курицей, луковые кольца и корневое пиво. Нэнси получила овощной бутерброд, картофельные чипсы и лимонад.

К сожалению, заказы не были доставлены должным образом.

Лори сказала: «Правильно только одно — и это определенно не вода».

Мэтт сказал: «И только один из моих пунктов верен!»

Нэнси сказала: «Только один из моих предметов неправильный.

Один из сотрудников сказал: «Я знаю, что бутерброд с курицей подходит к воде».

Другой сотрудник сказал: «Я знаю, что картофельные чипсы подходят к корневому пиву, но не к сэндвичу с рыбой».

Помогите менеджеру разобраться! Каждое утверждение верно. Кто что заказывал?

Проблема путаницы в фаст-фуде очень сложна! Бумага и карандаш могут оказаться большим подспорьем. Не сдавайся. Решение находится в конце статьи.

Головоломки.

Еще одна категория логических задач — классическая «головоломка». Логические дразнилки часто бывают сложными, потому что они полагаются на навык, называемый «нестандартным мышлением». Проблемы нестандартного мышления заставляют вас пересмотреть свои предположения по поводу вопроса. Вот классическая головоломка с нестандартным мышлением:

Что уникального в этом числе?

8,549,176,320

Спойлер! Мы собираемся дать ответ.Взгляните на минутку и посмотрите, сможете ли вы сначала во всем разобраться. Если вы застряли, взгляните на подсказки, прежде чем искать решение!

Подсказка № 1: Вы, наверное, заметили, что представлены все цифры (от 0 до 9). Хороший! И вы знаете, что это проблема латерального мышления (потому что это то, о чем этот раздел!) В задачах латерального мышления решение заключается в мышлении, выходящем за рамки предположений, которые вы привносите в проблему.

Поскольку эта логическая задача представляет вам числа, вы, вероятно, думаете, что должны использовать свои математические навыки, чтобы найти решение.Другими словами, как только ваш мозг видит числа, он быстро предполагает: «Ага! Ищите числовые отношения между числами! »

Суть проблем нестандартного мышления заключается в том, что решение останется неуловимым, пока вы остаетесь привязанными к предпосылкам, которые приносите с собой. Итак, есть ли другой способ думать о математической головоломке, которую вы видите?

Подсказка № 2: Что, если бы вам пришлось записать это число словами?

Подсказка № 3: Какова связь между первой буквой каждого слова, которое вы написали в предыдущей подсказке?

Поняли? Потрясающие! Это все цифры, записанные в виде слов в алфавитном порядке.

Сможете ли вы отгадать загадку тридцати центов?

Некоторые люди расстраиваются, когда находят решение проблемы нестандартного мышления. Это расстраивает, когда наши умственные ярлыки, которые обычно полезны, становятся источником наших неудач. Тем не менее, когда люди особенно искусны в мышлении, выходящем за рамки ограничений этих рамок, они часто становятся изобретательными предпринимателями и предприимчивыми изобретателями. Они развивают способность мыслить нестандартно и создавать революционные инновации.

Теперь, когда вы знаете, что этот раздел посвящен нестандартному мышлению и проверке ваших предположений, посмотрите, сможете ли вы решить следующую задачу самостоятельно. Вы можете начать с вопроса: «Какие предположения я делаю, которые могут быть перевернуты словами этого вопроса?» Давайте попробуем!

В руке у меня ровно две американские монеты. Вместе они составляют тридцать центов. Один из них — не пятак. Кто они такие?

Кажется невозможным, правда? Преимущество проблемы нестандартного мышления заключается в осознании того, что многие предположения о «невозможном» являются всего лишь функцией наших собственных ограничивающих убеждений.Если вы можете переосмыслить проблему по-новому, то, что когда-то считалось «невозможным», внезапно становится тривиальным!

Сдаться? Хотите ответ? Мы действительно сказали, что было две монеты, и действительно, одна из них — это не монета. Тот — это не никель — четверть. А другой ? Да, это никель. 😊

Тайна двух братьев. Невозможно?

На этом, посмотрим, сможете ли вы сломать свои предположения, чтобы найти ответ.

У женщины было два сына.Они родились в один и тот же час, в один и тот же день одного месяца одного года. Однако близнецами они не были. Как это возможно?

Хотя некоторые люди отвергают головоломки с латеральным мышлением как «настоящие логические головоломки», они имеют огромное значение в обучении детей (и взрослых) разнице между мышлением и тем, что ученые называют «метакогнитивным мышлением».

В типичной логической головоломке наша работа — думать. Но с боковыми рассуждениями наша работа — думать о мышлении.Когда в вопросе говорится, что сыновья не близнецы, мы всецело думаем о близнецах.

Это то «замкнутое мышление», которое мешает нам увидеть, что двух мальчиков, рожденных от одной матери в одно и то же время, условно называют близнецами, когда их всего двое. Но что, если трех братьев родились одновременно? В задаче все верно, но мы бы не стали называть этих мальчиков близнецами; мы бы назвали их тройняшками!

Отличительный признак боковых проблем: они всегда кажутся «невозможными» на основании информации.Однако невозможно сохранить логическую связь между нашими предположениями и простыми фактами вопроса.

Если вам нравится ощущение, что ваш разум скручивается в узлы, задачи с самореферентной информацией могут быть вашим любимым видом логической головоломки.

Головоломки со ссылками на себя, иногда называемые рекурсивными задачами, — это задачи, которые заставят вас почувствовать, как ваш мозг крутится в узлах. В этих задачах вопросы относятся к вопросам, которые относятся к вопросам… Вы понимаете.Это сложно. Давайте посмотрим на один классный, созданный инженером-программистом и создателем головоломок по совместительству Лаури Тервонен:

1. Каков ответ на второй вопрос?

A. A

B. B

C. C

D. D

2. Сколько правильных ответов в этом тесте — B?

A. 0

B. 1

C.2

D. 3

3. Есть ли вопрос с правильным ответом A?

A. Нет

B. Да, 1

C. Да, 2

D. Да, все 3

Забавьте учителей ссылочная головоломка.

Если вам понравилась эта задача, вот одна из самых коротких версий задачи со ссылками на себя. Этот замечательный, потому что его достаточно легко запомнить.Вы можете попытаться поставить в тупик своих учителей и родственников!

Угадайте следующие три буквы в серии GTNTL.

Возможно, вы сразу поняли ответ на этот вопрос, но если вы еще этого не сделали, не сдавайтесь слишком быстро! Еще одна причина работать над головоломками — стать более усидчивыми.

Все еще застрял? Подсказка: этот раздел посвящен самодостаточным головоломкам!

Вы уловили последний намек? Следующие три буквы в последовательности: «I, T, S.», Которые являются начальными буквами слов« в серии ». Начальные буквы слов в задаче: G для предположения, T для, N для следующего, и так далее, создавая последовательность: GTNTLITS…

Если вам нравятся эти примеры и вам интересна рекурсия, ознакомьтесь с этим отличная статья о рекурсии, языке и логике.

Есть другие типы? Да! Головоломки с логической сеткой, головоломки Какуро, головоломки Судоку, Ребус, Хитори, Ханджи, Нурикабе, Скользящая ссылка, Футосики, кроссворды, Магический квадрат, Механика, Криптос.И это только начало.

Вау. Это много разных видов, и это даже не касается всего! Компании и юридические школы даже используют логические головоломки при приеме на работу и приеме на работу, потому что они могут выявить когнитивные способности.

Если вам нравятся головоломки с логической сеткой, и вы научитесь их хорошо решать, попасть в лучшую юридическую школу будет намного проще. На LSAT, вступительном экзамене юридической школы, головоломки с логической сеткой составляют 23% вашего балла! Вот увлекательная статья о головоломках, которые ведущие технологические компании задают в своих интервью.

Как помочь ребенку разобраться в логических головоломках?

Лучший способ научиться решать головоломки — это разгадывать их! А как это сделать проще всего? Если вы хотите, чтобы у ваших детей был постоянный поток логического времени, свяжите его с другими привычками, которые у вас уже есть.

Вы всегда обедаете вместе после спортивной тренировки во вторник вечером?

Попробуйте найти этот фантастический источник задач «Московские головоломки: 359 математических увлечений» и каждую неделю задавайте один из вопросов за ужином.Тот, кто это выясняет первым, не должен мыть посуду!

Вот еще один пример использования связывания привычек для последовательной работы над логическими проблемами. Вы и ваш ребенок читаете книги вместе каждую ночь? На ночь или две в неделю добавляйте простую загадку с логическими рассуждениями, прежде чем читать!

Время в машине — еще одно отличное время для решения сложной головоломки. То, где вы их делаете, имеет меньшее значение, чем последовательность. Лучший способ быть последовательным — сделать интеллектуальные игры частью деятельности, которая уже происходит регулярно.Эту технику, называемую накоплением привычек, можно использовать для выработки любой здоровой привычки. Всего через несколько месяцев вы увидите, как у вашего ребенка возрастет способность решать проблемы. В течение сезона вы увидите, как улучшение аналитических навыков вашего ребенка положительно повлияет на его математические способности и любовь к предмету.

Настольные игры и головоломки для развития аналитического мышления.

Логические головоломки могут принимать разные формы. Даже знаменитый кубик Рубика — это логическая головоломка.Он относится к категории «механические головоломки». Такие занятия, как кубик Рубика и настольные игры, — отличный способ поощрить и развить аналитическое мышление, одновременно прекрасно проводя время! Настольная игра, такая как Splendor, которая основана на совершении сделок и оценке стоимости относительных предметов, отлично подходит для оттачивания навыков аналитического мышления.

Если вам нужна настоящая головоломка, а не соревновательная настольная игра, обратите внимание на линейку головоломок ThinkFun или головоломки Puzzle Baron’s Logic Grid.Учебная программа IMACS также отлично подходит для развития этих навыков. Фактически, один из разработчиков головоломок ThinkFun является выпускником IMACS. Вы можете прочитать историю Марка Энгельберга, чтобы узнать больше о том, как он придумал большую головоломку «Шоколадное исправление». Если вы хотите увидеть, как программа IMACS может сделать из вашего ребенка будущего конструктора головоломок, изобретателя, предпринимателя или отличного всестороннего мыслителя, приведите своего ребенка (виртуально!) На наш бесплатный класс оценивания. Мы поиграем с ними в математические игры в течение примерно тридцати минут и дадим вам бесплатный анализ размещения, чтобы вы знали, как лучше всего помочь вашему ребенку повысить уровень его математических способностей.

Собираем все вместе. Пять причин начать вместе с детьми решать логические головоломки.
  1. Логические задачи учат детей различным подходам к решению проблем. Такие методы, как устранение, обратная работа, «нестандартное мышление» и навыки критического мышления — все это подходы, которые получают тренировку и улучшаются.
  2. Активные мозги растут. Если мозг похож на мышцу, математические головоломки — это штанги. Часто тренируйте свой мозг, чтобы поддерживать форму и становиться сильнее.
  3. Интересная математика, которую делают инженеры-математики, изобретатели и математики, намного увлекательнее, чем механический счет и применение некоторых уравнений к набору чисел. Во многих отношениях логические головоломки так же важны, как и «школьная математика», в плане набора навыков, которые понадобятся вашим детям в будущем.
  4. Иногда одаренные дети учатся бояться математики. Иногда дети, которые испытывают трудности с изучением предмета, отключаются, потому что думают, что это скучно. Однако часто это происходит потому, что их единственное воздействие — запоминание формулы или заполнение рабочего листа.Головоломки — отличный способ помочь детям, которые бросают учебу в школе, увидеть, насколько прекрасна и увлекательна математика.
  5. Логические головоломки — это замаскированные словесные задачи. «Нет! Это не страшная проблема со словами », — стонут многие дети, когда видят стену текста в учебнике по математике. Но для успеха в математике необходимы навыки понимания прочитанного. Головоломки чаще всего представляют собой задачи со словами, поэтому дети учатся получать удовольствие от формата, а не бояться его.

Большой фантастический секрет, который заставит ваших детей полюбить логические головоломки.

Иногда сложно увлечь детей тем, чего нет на экране. Мы получим это; мы тоже родители. Но за тридцать лет обучения детей навыкам логического мышления мы придумали один верный трюк, который заставит детей увлечься усердной работой над сложными проблемами: сказать им ответ.

Эй! Пока не нажимайте кнопку возврата! Вот как ответ помогает им взволноваться. Получите несколько логических головоломок; отличный источник — шоу NPR «Car Talk», которое заканчивает каждую серию блестящей головной болью.Пусть ваш ребенок покажет вам задачу. Переверни сценарий! Позвольте им наблюдать, как вы боретесь с этим, работаете над этим и, самое главное, получайте от этого удовольствие. Эти головоломки — это забавные головоломки, но все равно решите их. Когда вы попробуете еще один (может быть, через несколько дней или неделю), настаивайте на том, чтобы вы сначала попробовали его. Неделя третья, скажите, что вы хотите попробовать первым, но вы готовы, отпустить их первыми.

Моделируя, насколько весело решать головоломки, вы даете им повод попробовать.Этот принцип работает и в наших классах. Наши учителя так весело преподают математику ученикам IMACS, что дети с нетерпением ждут этого. Если вы еще не посещали наши занятия, вы можете узнать больше о детях, которые их посещали; если вы хотите сами увидеть наши занятия, подпишитесь на бесплатную пробную версию.

Спасибо за чтение! Вот вам еще одна забавная задача.

У отца Петра пятеро сыновей. Имена четырех сыновей — Фефе, Фифи, Фафа и Фуфу соответственно.Как зовут пятого сына?

Если они вам понравились, вам понравится IMACS. Попробуйте бесплатное занятие по размещению и посмотрите, насколько увлекательны математические рассуждения.

Ответы:

Загадки для игры слов:

Что намокнет? Полотенце.

Что можно запустить? Река

Я невесомая? Отверстие

Случай с пролитым какао:

Решение: Аврора

Пояснение:

Утверждения 1 (Элизабет) и 3 (Аврора) по сути говорят об одном и том же.Оба эти утверждения не могут быть правдой, поэтому Элизабет и Аврора должны лгать. Следовательно, Ророна должен говорить правду. Поскольку мы знаем, что какао пролила не Ророна, это, должно быть, Аврора, поскольку она единственная сестра, помимо Ророны, в имени которой есть буквы, которые могут «рычать».

Фиаско с фастфудом:

Решение:

Лори заказала бутерброд с рыбой, луковые кольца и лимонад.

Мэтт заказал бутерброд с курицей, картофель фри и воду.

Нэнси заказала овощной бутерброд, картофельные чипсы и корневое пиво.

Пояснение:

Поскольку картофельные чипсы идут с пивом, либо сторона Нэнси неправильная, либо ее напиток неправильный. Итак, ее вегетарианский бутерброд правильный. Лори не получила сэндвич с курицей (потому что он идет с водой, которую она не получила). Отсюда следует, что:

Лори заказала бутерброд с рыбой, ________ и ________.

Мэтт заказал сэндвич с курицей, ________ и воду.

Нэнси заказала вегетарианский бутерброд, ________ и ________.

Поскольку только один из пунктов Мэтта был правильным, он не получил луковые кольца. Поскольку чипсы идут с пивом, Мэтт чипсов не получил. Итак, он получил картошку фри.

Решение головоломки со ссылками на себя:
  1. (A)
  2. (A)
  3. (C)

А как насчет пятерых сыновей? Пятый сын отца Петра — Петр!

Логические головоломки с ответами по математике — Математические головоломки с ответами

Логические головоломки являются неотъемлемой частью банковских PO, SSC, государственных экзаменов и многих вступительных испытаний.Вопросы, основанные на головоломках, предназначены для проверки ваших аналитических, дедуктивных способностей и способностей к решению проблем. Здесь мы собрали ряд вопросов и ответов на различные логические головоломки в порядке сложности. Вы научитесь решать аналитические головоломки и вопросы логического мышления. Эта статья содержит 10 вопросов и ответов на головоломки среднего уровня.

Решите заданные вопросы и проверьте свои умственные способности:

Q.1. Внимательно прочтите следующую информацию и ответьте на вопросы, заданные ниже.
  1. P, Q, R, S и T — пять друзей.
  2. Q старше Т, но не такой высокий, как R.
  3. R младше P и выше S и T.
  4. P выше S и не самый высокий, но моложе T.
  5. S старше P, но самый короткий в группе.

Кто из следующих самый высокий?

Решение и объяснение

С точки зрения возраста, мы имеем T

Q.2. Внимательно прочтите следующую информацию и ответьте на вопросы, заданные ниже.

P, Q, R, S и T — пять друзей.

Q старше T, но не такой высокий, как R.

R младше P и выше S и T.

P выше S, но моложе T.

S старше P, но самый короткий в группе.

Кто из следующих старший?

Какая из следующих пар учеников старше S?

  1. QP
  2. QR
  3. QT
  4. TP
  5. Не определяется
Решение и объяснение

С точки зрения возраста, мы имеем T

Q.3. Мужчина смотрит на картину в доме своей матери, единственного брата, племянника. Кто-то проходит мимо этого человека и спрашивает: «На кого ты смотришь?» Мужчина ответил: «Я смотрю на фотографии единственной сестры моего сына, дочери деда по материнской линии, мужа, отца сына. Кроме того, ни у моей жены, ни у меня нет братьев и сестер ». В чьем доме этот мужчина и на кого он смотрит?

Решение и объяснение

Мужчина находится в собственном доме и смотрит на свою фотографию.

Племянник единственного брата матери человека — это сам мужчина, поскольку у мужчины нет братьев или сестер. Таким образом, он в своем доме.

Единственная сестра сына мужчины = его дочь

гарант дочери по материнской линии = отец жены

по материнской линии и дочка отца = его жена (так как у его жены нет братьев и сестер)

Отец сына мужа жены — он сам.

Решение и объяснение

По возрасту E

Итак, мы имеем: C

По высоте: B

Q.5. Рамеш решил, что Пунам, ее дочь, выйдет замуж за Раджата. Пунам отказался и несколько дней плакал, ничего не ел. Рамеш, чтобы решить эту проблему, решил, что ей следует достать из сумки маленький шарик. Если Пунам возьмет синий шарик, ей придется выйти замуж за Раджата, если она возьмет красный шарик, ей не придется. Несколько дней спустя Раджат, Рамеш и Пунам были вместе на дороге к замку, вымощенной маленькими синими и красными шариками.Рамеш просит Раджата взять два шарика и положить их в мешок. Пунам увидел, что Раджат вставляет два синих шарика. Она думает об этом сказать, но тогда, скорее всего, процедуру придется начинать заново. Внезапно ей приходит в голову отличная идея. Через несколько минут решено, что Пунам не обязательно выходить замуж за Раджата. Что она сделала?

Решение и объяснение

Она просто вынула один маленький шарик, но тут же бросила его на дорожку вместе со всеми остальными шариками. Затем она утверждает, что шарик, который она взяла, должен быть противоположным тому, который остался в сумке.Итак, поскольку в шляпе было два синих шарика, можно сделать вывод, что она взяла красный шарик. Так что Пунам, к счастью, не обязательно выходить замуж за Раджата.

Q.6. После долгого пути водитель грузовика оказывается перед туннелем. Он не может проехать, не повредив свой грузовик, потому что высота туннеля составляет 4,00 метра, а высота его грузовика — 4,01 метра. Водитель хотел бы проехать через туннель, потому что в обратном направлении у него уйдет не менее 5 часов.Внезапно ему приходит в голову отличная идея, он немедленно начинает что-то делать, и через 10 минут он снова возвращается в нужное русло на другой стороне туннеля. В чем заключалась отличная идея водителя грузовика?

Решение и объяснение

Водитель удалил воздух из шин.

Q.7. Раджа Махиндер Пратап Сингх решил построить стену, чтобы защитить свое королевство от вражеских атак. Стена должна была быть 5 метров в толщину и 13 метров в высоту. Граница страны имеет протяженность 1350 километров.Если один кубический метр камня весит около 2660 кг, насколько тяжелее станет земля, построив эту стену?

Решение и объяснение

Очевидно, камни исходят из земли; масса земли, конечно, не изменится, построив такую ​​большую стену.

Q.8. У Ануджа 22 пары черных перчаток и 22 пары синих перчаток. Он хранит их все в одном ящике. Какое минимальное количество перчаток необходимо для создания подходящей пары?

Решение и объяснение

В худшем случае 44 перчатки одних, скажем, левой.Теперь 45-я перчатка будет правой рукой и составит пару.

Q.9. Сколько плиток нужно, чтобы закончить комнату из плитки?

Решение и объяснение

Только один — «последний».

Q.10. Рошни и Пракаш и два их сына Сан и Рэй подошли к реке и нашли лодку. Он может нести Солнце и Рэй, Рошни или Пракаш одновременно. Оба сына хороши в гребле, но как четверо могут добраться до другого берега реки? & Сколько раз лодка будет переходить с одного борта на другой.

Решение и объяснение

Сыновья гребут. Один возвращается. Рошни подходит, и один сын возвращается. Оба сына снова гребут, а один возвращается. Потом Пракаш переправляется, и сын возвращается. Оба сына снова гребут.

Всего 9 раз произойдет движение лодки

Самая сложная логическая головоломка

Самая сложная логическая головоломка — загадка логиков Раймонда Смолляна и Джон Маккарти, прославившийся в 1990-х годах.

Кто истинен, кто ложен, а кто случайен?

Представьте, что есть три оракула: Истинный, Ложный и Случайный. Истина всегда говорит правду, Ложь всегда лжет и Случайно говорит случайным образом правда или ложь. Ваша задача — выяснить, что это за оракул, задав им несколько вопросов. Да, без вопросов. Но есть дополнительная трудность: оракулы ответят своим мистическим язык, говоря «DA» или «BAL», и вы не знаете, какое из этих слов означает да и что означает нет.

Удивительно, но трех вопросов достаточно, чтобы разгадать загадку.

Прежде чем вы начнете размышлять об этом, вот три вещи, которые важно знать:

  1. Случайный оракул решит, солгать или нет, или нет, тайно подбросив монетку.
  2. Вы можете задать любой вопрос да / нет любому оракулу по вашему выбору.
  3. Необязательно задавать сразу все вопросы. Фактически, вы можете использовать предыдущие ответы, чтобы адаптировать свои вопросы и выбрать оракула, которому вы их задаете.

Если вы не можете найти решение, не отчаивайтесь. В конце концов, это называется самой сложной логической головоломкой на свете. Вот общая идея решения. В первом вопросе вы определяете оракул X, который не является случайным. Затем вы задаете два дополнительных вопроса X. Второй вопрос определяет, является ли X истинным или ложным, а третий вопрос определяет два других оракула.

Если и только если

В решении используется логическая концепция , если и только если , который работает следующим образом: даны два оператора A и B, оператор A тогда и только тогда, когда B истинно, когда A и B оба истинны или оба ложны, в противном случае это ложь.Это означает, что выписка

2 + 2 = 4 тогда и только тогда, когда Луна сделано из сыра

ложно, потому что один компонент — истина, а другой — ложь. С другой стороны, заявление

2 + 2 = 5 тогда и только тогда, когда Луна изготовлен из сыра

верно, потому что оба компонента ложны. Эти примеры не совсем соответствуют нашей интуитивной интерпретации того, что может означать «если и только если», но постарайтесь не беспокоиться об этом. Просто помните логическое определение, данное выше.( Если и только если является отрицанием того, что называется XOR, о чем вы можете прочитать здесь.)

Пуск простой

А теперь вернемся к нашей загадке и начнем с упрощения. Предположим, что DA означает «да», а BAL означает «нет».

Сначала предположим, что вы уже задали первый вопрос какому-то оракулу, который определил другой оракул X, который не является случайным. Можете ли вы тогда придумать простой вопрос, который раскроет, кто такой X? После этого вы можете придумать простой вопрос для X, который раскроет, кто такие два других оракула?

Если нет, то вот ответы.Чтобы узнать, истинно ли X или нет, вы просто задаете вопрос, на который вы уже знаете ответ, например:

2 плюс 2 равно 4?

Чтобы идентифицировать двух других оракулов, вы можете спросить X:

Случайно ли этот оракул (указывая на оракула, который вы задали первым вопросом)?

Поскольку вы знаете, является ли X истинным или ложным, вы можете определить, является ли оракул, который вы задали первым вопросом, случайным или нет. Путем исключения вы также идентифицируете третьего оракула.

Первый вопрос

Снова предположим, что DA означает «да», а BAL означает «нет» (мы позаботимся о языковом вопросе позже), и давайте обратимся к первому важному вопросу, который является самым сложным. Выбирать любой оракул O и спросите:

Верны ли вы тогда и только тогда, когда этот оракул (где вы указываете на одного оракула P, отличный от O) является случайным?

Луна не из сыра, и у нее нет колец.

Предположим, ответ, который вы получили от O, — «да».Если O истинно, то оракул P должен быть случайным. Это потому, что заявление

O истинно тогда и только тогда, когда P случайное

истинно, если оба компонента истинны или оба компонента ложны.

Если ответ «да» и O — ложь, то истинным ответом должно быть «нет, утверждение« O истинно тогда и только тогда, когда P является случайным »- ложно». Поскольку первая часть утверждения «O истинно» является ложной, вторая часть утверждения «P является случайной» должна быть истинной (из определения тогда и только тогда, когда ).

Итак, независимо от того, истинно ли O или нет, ответ «да» говорит нам, что P является случайным, что означает, что третий оракул, назовем его Q, не является случайным.

Что говорит нам ответ «да», если O является случайным? Что ж, это не говорит нам с уверенностью, что такое P, но это не имеет значения. В этом случае, поскольку O является случайным, третий оракул Q определенно не является случайным.

Таким образом, ответ «да» с уверенностью говорит нам, что третий оракул Q не является случайным, независимо от того, что такое O.

Что, если O ответит «нет»? Тогда, если O истинно, это означает, что P не является случайным. Если O — False, то истинным ответом должно было быть «да, утверждение верно». С первой части выписки

O истинно тогда и только тогда, когда P случайное

в этом случае ложно, это означает, что вторая часть «P is Random» также ложна.

Итак, независимо от того, истинно ли O или ложно, ответ «нет» говорит нам, что P не является случайным.

Что говорит нам ответ «нет», если O является случайным? Что ж, это говорит нам, что P не может быть случайным, потому что O уже есть.

Таким образом, ответ «нет» с уверенностью говорит нам, что P не является случайным, независимо от того, что такое O.

В обоих случаях, независимо от того, отвечает ли O «да» или «нет», мы определили, что один оракул не является случайным. Если O отвечает «да», то третий оракул Q не является случайным. И если O отвечает «нет», то оракул P не случайный. Это идентифицирует наш неслучайный оракул X сверху. Теперь мы можем перейти к двум другим вопросам, как описано, чтобы разгадать загадку. На диаграмме ниже показаны ответы на три вопроса.

Эта диаграмма показывает, как различные ответы на три вопроса позволяют нам идентифицировать все три оракула. Здесь T означает True, F означает False, а R означает Random.

Обратите внимание, что роль тогда и только тогда, когда здесь, заключалась в том, чтобы получить достоверность из ответов O, не зная, является ли O Истинным, Ложным или Случайным.

DA и BAL

Теперь рассмотрим исходную загадку, в которой вы не знаете значения DA и БАЛ. Здесь нам нужно ввести еще один уровень , если и только если , что позволит нам получить ту же информацию, что и раньше, без знания того, какой из DA и BAL означает «да», а что означает «нет».А именно, добавляя «если и только если DA означает« да »в конце всех трех вопросов, получается, что мы можем интерпретировать ответ DA как положительный ответ на исходный вопрос, а ответ BAL как отрицательный ответ на вопрос оригинальный вопрос. В следующем разделе мы увидим пример этого, а теперь мы собираемся сформулировать общее решение загадки.

Вы начинаете с того, что выбираете оракул O и спрашиваете его, указывая на другой оракул P,

Вы истинны тогда и только тогда, когда P является случайным, тогда и только тогда, когда DA означает да?

Это утверждение верно, если два утверждения

Верны ли вы тогда и только тогда, когда P случайное

и

DA означает да

оба истинны или оба ложны, в противном случае это ложь.

Как и раньше, этот вопрос идентифицирует оракул X, отличный от O, который не является случайным.

Затем вы спрашиваете у X:

2 плюс 2 равно 4 тогда и только тогда, когда DA означает да?

Это скажет вам, истинно ли X или ложно. Наконец вы спрашиваете X:

Является ли этот оракул (указывая на оракула О, которому вы задали первый вопрос) Случайным, если и только если DA означает да?

Как и раньше, это скажет вам, является ли O случайным или нет, и, поскольку вы уже знаете, что такое X, он решает загадку.

В самом деле?

Чтобы убедить вас в том, что это работает, давайте рассмотрим последний вопрос. К этому моменту вы уже знаете, истинно ли X или нет. Предположим, что X истинно. Вы спрашиваете X об О, оракуле, которому вы задали первый вопрос.

На этой картине Джона Коллиера изображена Пифия, оракул Дельфийский (который мог бы сказать больше, чем просто DA и BAL).

Предположим, что X отвечает DA, а DA означает «да». Тогда O является случайным. Теперь предположим, что DA означает «нет». Тогда O также является случайным.Это потому, что в этом случае оператор

DA означает да

ложно, поэтому, поскольку X сказал DA, что означает «нет», утверждение

O случайно

верно.

Что делать, если X говорит BAL, а BAL означает «да». Тогда O не является случайным. Это потому, что в этом случае оператор

DA означает да

ложно, поэтому, поскольку X сказал BAL, что означает «да», утверждение

O случайно

также должно быть ложным.

Аналогично, если BAL означает «нет», то O также не является случайным. Это потому, что в этом случае оператор

DA означает да

верно, поэтому, поскольку X сказал BAL, что означает «нет», утверждение

O случайно

должно быть ложным.

Мы только что показали, что если X истинно, то ответ DA сообщает нам, что O является случайным, а ответ BAL говорит нам, что O не является случайным, даже если мы не знаем, что означают DA и BAL. .

Очень похожий аргумент показывает, что, когда X является ложным, тогда ответ DA означает, что O не является случайным, а ответ BAL означает, что O является случайным. Таким образом, мы можем определить, является ли O случайным или нет, из ответа X, не зная, какой из DAL и BAL означает «да», а что означает «нет».

Также можно показать, что ответ на первый вопрос идентифицирует оракул X, который не является случайным, а ответ на второй вопрос определяет, истинно ли X или ложно, даже если вы не знаете, что из DAL и BAL означает «да», что означает «нет».Мы оставим это вам в качестве упражнения.

Еще одна головоломка для вас

В телешоу вам показывают два конверта. Если вы откроете правильный конверт, вы выиграете огромную сумму. приз, а если откроешь не тот, то ничего не выиграешь. Есть два человека и ты может задать одному из них вопрос «да / нет». Один человек скажет правду, а другой — ложь (и вы не знаете, кто говорит правду, а кто лжец). Какой вопрос может вы спросите, чтобы идентифицировать конверты?

Вы можете увидеть решение здесь.


Об авторе

Антонелла Перукка — профессор математики и ее дидактики в Люксембургском университете. Она занимается теорией чисел и изобретает математические экспонаты. Чтобы узнать больше, изучите ее веб-страницу.


Математические пазлы

  • Есть большое удовлетворение, что можно получить, решив математическую головоломку. Здесь очень много головоломки на этой странице, все с математической связью, которые просто ждет своего решения.Вы можете заработать Transum Trophies, решая головоломки. решать.

  • Номера самолетов

    Расположите числа на плоской сетке, чтобы получить заданные суммы

  • Криптографические

    Заполните квадраты в соответствии с подсказками, указанными в цепочке чисел для каждой строки и столбца.

  • Без ведома

    Некоторые головоломки в виде сетки с картинками, которые можно решить с помощью одновременных уравнений.

  • Группы по четыре человека

    Упрощенная математическая версия задачи, показанной в британской телепрограмме Only Connect. Найдите связи между терминами.

  • Частичные пирамиды

    Подсчитайте недостающие числа в этих частично завершенных головоломках-пирамидах.

  • Пазл-пирамида

    Числа в кубиках можно найти, сложив два кубика сразу под ними вместе.Сможете ли вы достичь поставленной цели?

  • Головоломка Сломанная шахматная доска

    Шахматная доска разбита на 13 частей. Вы можете собрать его обратно?

  • Рассечение пончиков

    Головоломка, в которой нужно найти четыре различных способа получить 900 путем умножения трех разных чисел.

  • Power Shift

    Расположите указанные числа как основания и индексы в трехчленной сумме, чтобы получить целевую сумму.

  • Удовлетворение

    Это довольно сложная головоломка с группировкой чисел, требующая знания простых, квадратных и треугольных чисел.

  • Squorder

    Версия Transum традиционной головоломки с раздвижной плиткой.

  • Найти шахту

    Найдите, где спрятаны мины, не наступая на них.

  • Пентамино

    Расположите двенадцать пентамино по контуру прямоугольника.

  • Номер Лобзик

    Онлайн, интерактивные пазлы из сеток чисел.

  • Головоломка с римскими цифрами

    Интерактивная онлайн-головоломка из сетки римских цифр.

  • Лобзик Magic Square

    Интерактивные головоломки из четырех магических квадратов.

  • плюс

    Пазл с семью уровнями сложности.

  • Площадь лабиринта

    Используйте свои знания о прямоугольных областях, чтобы вычислить недостающие размеры этих составных диаграмм.

  • Пазл о парковке

    Можете ли вы вывести свою машину из переполненной парковки, переместив другие машины вперед или назад?

  • Только одна цифра

    Найдите выражения с использованием только одной цифры, которые соответствуют заданным целям.

  • Центровыражение

    Расположите числа от 1 до 9, чтобы получить выражение со значением 100.

  • Идеальный магический квадрат

    Расположите шестнадцать чисел на сетке четыре на четыре так, чтобы группы из четырех чисел в шаблоне составляли одну и ту же сумму.

  • Octagram Star

    Расположите шестнадцать чисел в октаграмме так, чтобы сумма чисел в каждой строке равнялась одинаковой сумме.

  • Гексаграмма Звезда

    Расположите двенадцать чисел на гексаграмме так, чтобы сумма чисел в каждой строке равнялась одинаковой сумме.

  • Треугольная гексаграмма

    Расположите двенадцать чисел в треугольниках гексаграммы так, чтобы сумма чисел в каждой строке из пяти треугольников равнялась одинаковой сумме.

  • Awe-Sum

    Расположите указанные цифры так, чтобы получилось шесть 3-значных чисел, которые прекрасно сочетаются друг с другом.

  • Целевые продукты

    Расположите пронумерованные футбольные мячи на стойках ворот, чтобы получилось три одинаковых продукта с 3 номерами.

  • Обхват

    Довольно сложная головоломка с числами, состоящими из дробей.

  • Магический квадрат

    Каждая строка, столбец и диагональ должны давать одинаковую сумму.

  • Unmagic Square

    Как магический квадрат, но все суммы должны быть разными.

  • Туннель с одним факелом

    Решите задачу прохождения четырех человек через туннель с одним фонариком за минимальное время.

  • Разлученные близнецы

    Можете ли вы найти шестизначное число, содержащее по две цифры каждая из цифр с первой по три, которое подчиняется данным правилам?

  • Не отрывая карандаш

    Можете ли вы нарисовать эти схемы, не отрывая карандаш от бумаги? Это интерактивная версия традиционной головоломки.

  • Переход через реки

    Традиционное испытание на переходе через реки.Сможете ли вы сделать это за наименьшее количество ходов?

  • Digivide

    Расставьте числа от 1 до 6 в полях, чтобы расчет деления был правильным.

  • Разведчики в лодках

    Составьте расписание для скаутов, чтобы они путешествовали на лодках, чтобы они были с разными людьми каждый день.

  • Quad Area

    Вычислите площади всех возможных четырехугольников, которые можно построить, соединив точки на этой сетке.

  • Четыре суммы

    Расположите плитки с указанными числами, чтобы получилось два двузначных числа, которые в сумме дают заданное общее количество.

  • Путь Головоломка

    Отличная головоломка, требующая от вас использовать все карты, чтобы создать непрерывную красную линию от начала до конца.

  • Лимонный закон

    Измените числа на яблоках так, чтобы число на лимоне равнялось заданной сумме.

  • Головоломка с расписанием

    Составьте расписание 24-часового дартс-марафона, которое учтет все запросы и сделает всех счастливыми.

  • Интернет-экстрасенс

    Позвольте экстрасенсу прочитать карты и волшебным образом открыть число, которое вы тайно выбрали. Какая математика заставляет этот трюк работать?

  • Numskull

    Интерактивная логическая головоломка на основе чисел, генерируемая случайным образом, для развития навыков счета.

  • Addle

    Расставьте числа от 1 до 14 в полях, чтобы суммы были правильными. Как быстро ты сможешь это сделать?

  • Суко Судзико

    Интерактивные логические головоломки на основе чисел, похожие на те, что публикуются в ежедневных газетах.

  • Пазлы с латинскими квадратами

    Расположите указанные цифры так, чтобы получился латинский квадрат с заданными результатами вычисления строки и столбца.

  • Части многоугольника

    Расставьте девять частей пазла на сетке, чтобы образовать разные многоугольники.

  • Пу Вианг

    Задача встречного обмена с наименьшим количеством ходов, изобретенная в северном Таиланде.

  • Разочарование

    Логическая задача, требующая стратегии для обновления каждого числа в сетке.

  • Prime Square

    Перетащите числа в красные ячейки так, чтобы сумма трех чисел в каждой строке и каждом столбце была простым числом.

  • Смеси

    Разложите карточки, чтобы составить верное математическое утверждение.

  • Масленица

    Перемешивайте блины, пока они не будут аккуратно сложены по размеру.Найдите, как это сделать, используя наименьшее количество ходов.

  • Поиск слов

    Найдите математические слова в сетке букв.

  • Вектор Менты

    Помогите полицейским поймать грабителей, найдя векторы, которые положат конец погоне.

  • Трафальгарская площадь

    Решите числовые головоломки, нарисованные на тротуаре Трафальгарской площади в Лондоне.

  • Triside Totals

    Расположите цифры от 1 до 9 в треугольнике так, чтобы сумма чисел на каждой стороне была равна заданной сумме.

  • Множество

    Расположите указанные цифры так, чтобы получилось три числа так, чтобы третье было произведением первого и второго.

  • Двойной тройной

    Расположите цифры так, чтобы получилось три трехзначных числа так, чтобы второе было в два раза больше первого, а третье было в три раза больше первого.

  • Не слишком близко

    Студенты с номерами от 1 до 8 должны сесть на стулья так, чтобы никакие два последовательно пронумерованных студента не сидели рядом друг с другом.

  • Инструменты

    Сколько разных способов можно расположить числа, чтобы получить одинаковые итоги?

  • Вид из окна

    Перетащите 20 цветов в сад, чтобы из каждого окна дома было видно по 9 цветов.

  • Фигуры в звездах

    Соединяйте звезды, чтобы найти скрытые правильные многоугольники.

  • Удовлетворительно

    Поместите девять чисел в таблицу так, чтобы они соответствовали заголовкам строк и столбцов о свойствах чисел.

  • T Пазл

    Используйте части головоломки T, чтобы вписаться в предоставленные контуры.Интерактивная задача перетаскивания, поворота и падения.

  • Маневровые пазлы

    Как можно быстрее переместите трамваи на указанные стоянки на маневровой дворе.

  • Пазл Magic Square

    Найдите все возможные способы получения магической суммы из чисел в этом магическом квадрате четыре на четыре.

  • Cubical Net Challenge

    Найдите все способы раскрашивания граней кубиков, используя только два цвета.

  • Девять цифр

    Расположите указанные цифры так, чтобы получилось три числа так, чтобы два из них в сумме давали третье.

  • Tangram Стол

    Используйте части пазла танграм, чтобы сделать основные формы, затем заполните таблицу, показывающую, какие формы возможны.

  • слов калькулятора

    Переверните калькулятор, чтобы составить слова из ответов на эти вопросы.

  • Гласные

    Из математических слов было удалено

    гласных. Вы их узнаете?

  • Палочки для захвата

    Если бы вы поднимали палки из этой кучи и всегда снимали верхнюю палку, какой расчет вы бы создали?

  • Разделительный

    Расположите цифры с первого по девять в отведенных местах, чтобы произвести два вычисления деления, кратные трем.

  • Go Рисунок

    Расположите цифры от одного до девяти, чтобы четыре вычисления были правильными.

  • Олимпийские кольца

    Разместите цифры от одной до девяти в каждом из регионов, образованных олимпийскими кольцами, так, чтобы сумма чисел в каждом кольце была одинаковой.

  • Арифмагоны

    Найдите недостающие числа в этих треугольных головоломках с функцией самопроверки и откройте для себя чудеса этих увлекательных построек.

  • Пазлы для стен

    Разделите сетку на прямоугольные части так, чтобы площадь каждой части была такой же, как и число, которое она содержит.

  • Рождественские украшения

    Практическое задание, требующее от студентов сложить рождественские украшения в квадратную коробку.

  • Pentadd Quiz

    Найдите пять чисел, которые при попарном сложении или умножении дают заданные суммы или произведения.

  • Прайм Лабиринт

    Найдите путь к центру лабиринта, двигаясь по простым числам.

  • Жестокий

    Можете ли вы расположить семь жетонов на сетке, несмотря на их агрессивное поведение?

  • Математический кроссворд

    Интерактивный математический кроссворд для вас онлайн.Найдите недостающие слова из заданных подсказок.

  • Истерика

    Игра, головоломка и задача, в которой фишки размещаются по углам квадрата на сетке.

  • Подлецы и негодяи

    Расставьте негодяев и негодяев на стульях так, чтобы числа любых двоих, сидящих рядом друг с другом, составляли простое число.

  • Нет партнера

    Найдите, какие числа в данном списке не сочетаются с другими числами в списке для получения заданной суммы.

  • Трижды

    Можете ли вы расположить все фишки на сетке в 10 линий по три фишки?

  • Домино Пазл

    Разложите домино на семи квадратах. Количество точек на каждой стороне квадрата должно быть равно числу в середине

    .
  • Флер-де-Лис

    Щелкните на шести геральдических лилиях, чтобы оставить четное число в каждой строке и столбце.

  • Головоломка Миллера

    Это интерактивная версия головоломки, описанной Генри Эрнестом Дудени в «Кентерберийских головоломках

    ».
  • Восхитительно делимый

    Расположите цифры от одного до девяти, чтобы получилось число, которое делится описанным способом.

  • Девять Девять Девять

    Используйте цифры от 1 до 9, чтобы составить три трехзначных числа, которые в сумме дают 999.

  • Spinsum

    Расположите числа на квадратах так, чтобы суммы по каждой линии из трех квадратов были равны.

  • Паттерны

    Интерактивное задание, в котором вам предстоит воспроизвести узор из цветных квадратов в соответствии с заданными подсказками.

  • Brainbox

    Головоломка, требующая расположения чисел на функциональных машинах, чтобы связать заданные входные числа с правильными выходными.

  • Zygo

    Интерактивная логическая головоломка на основе чисел, генерируемая случайным образом, для развития навыков счета.

  • Судоку онлайн

    Интерактивная онлайн-версия популярной головоломки с числами.

  • Пазл Cube Net

    Куб-головоломка с беспорядочными движущимися блоками показан как сеть.Вы можете это решить?

  • Где Валлаби?

    Найдите спрятанного валлаби, используя подсказки в выбранных координатах.

  • Кувшины

    Можно ли приготовить 4 литра, если у вас есть только кувшины на 7 и 5 литров?

  • Взломщик кода

    Взломайте код, заменив зашифрованные буквы в данном тексте.Есть много подсказок о методах взлома кода.

  • Сколько квадратов? 2

    Сколько разных наборов из четырех точек можно соединить в квадрат?

  • Ханойская башня

    Перемещайте части башни с места на место за минимальное количество ходов.

  • Людикросс

    Расположите указанные числа на кресте так, чтобы сумма чисел на обеих диагоналях была одинаковой.

  • Самый крупный продукт

    Операция перетаскивания мышью, в которой вам нужно расположить цифры для получения максимально возможного продукта.

  • Подсказка Судоку

    Другой способ решить головоломку Судоку с подсказками, доступными на каждом этапе.

  • Пазлы для начинающих

    На сайте Transum есть еще много головоломок.


Следующие головоломки взяты из ежемесячных информационных бюллетеней и подкастов Transum.

Половина среднего угла

Найдите размер угла в точке A, если он составляет половину среднего значения углов в точках B и C.

Newsletter Podcast

Продовольственная инфляция

Стоимость еды увеличивается на 4% до целого числа фунтов.

Подкаст информационного бюллетеня

Крис и его непредсказуемые дети

Подумайте о головоломке с числами с непредсказуемым элементом.

Подкаст информационного бюллетеня

Кейт и Кэт

Определите возраст Кейта и Кэт по предоставленным подсказкам.

Подкаст информационного бюллетеня

Двуглавый и синий

У скольких меньших кубиков, составляющих куб 3×3, две грани окрашены в синий цвет?

Подкаст информационного бюллетеня

Санта крутой

Какая температура указывается с одним и тем же числом в градусах Цельсия и Фаренгейта?

Подкаст информационного бюллетеня

Три тройки троек, трижды

Простой вопрос, на который ошиблись семьдесят два процента группы людей.

Подкаст информационного бюллетеня

Поезда вместе

Как далеко друг от друга будут находиться поезда за полчаса до встречи?

Подкаст информационного бюллетеня

Шесть веревок

Могут ли пираты завоевать свободу, связав шесть кусков веревки в одну большую петлю?

Подкаст информационного бюллетеня

Богдан и Уолли

В каком порядке вы должны играть в Ultimate Noughts and Crosses против Уолли и Богдана?

Подкаст информационного бюллетеня

математических дней

Какая дата следующего дня теоремы Пифагора?

Подкаст информационного бюллетеня

Eva’s Eggs and Fickle Fractions

Сколько яиц Ева вывела на рынок, чтобы сделать эти странные половинки продаж?

Подкаст информационного бюллетеня

Кормление дураков и лошадей

Как долго лошадь будет кормить лошадей после того, как некоторые из них покинули конюшню?

Подкаст информационного бюллетеня

Балансировка воздушных шаров

Сколько воздушных шаров Джейми должен дать Бену, чтобы сбалансировать уравнение воздушного шара?

Подкаст информационного бюллетеня

Головоломка от Карла

Головоломка Карла представляет собой результат деления произведения возраста его родителей на удвоение его собственного возраста.

Подкаст информационного бюллетеня

Коэффициенты от событий до 2020 года

Вычтите сумму нечетных чисел меньше 2020 из суммы четных чисел меньше 2020 года.

Newsletter Podcast

Ноэль в Лапландии

Определите продолжительность поездки Ноэля в Лапландию, учитывая подробности погодных условий.

Подкаст информационного бюллетеня

Несколько остатков

Какое второе наименьшее число такое, что при делении на 5 остаток равен 4, а при делении на 7 остаток равен 6?

Подкаст информационного бюллетеня

Автобусы до Кембриджа

Можете ли вы выяснить, на каком автобусе я ехал, когда ехал в Кембридж?

Подкаст информационного бюллетеня

Дети Перси Кода

Перси Код говорил о своих детях.Можете ли вы определить их возраст с учетом соотношений?

Подкаст информационного бюллетеня

Морковьеды

Определите, сколько животных каждого вида находится в поле, по их морковные привычки.

Новостная рассылка Подкаст

Площадь Красной Стрелки

Какую часть квадрата закрывает красная стрелка?

Новостная рассылка Подкаст

Пэтси любит Перси

Странный случай с Перси, который, кажется, очень быстро стареет

Новостная рассылка Подкаст

Обезьяны, котята и собаки

Кто, скорее всего, сможет вычислить квадратный корень из 121?

Новостная рассылка Подкаст

Покормить лошадей

Как Долго ли хватит оставшегося корма для не проданных лошадей?

Новостная рассылка Подкаст

Странное дополнение

Если Я начинаю с пяти и добавляю шесть, я получаю одиннадцать, но если я начинаю с шести и добавляю семь Я получаю один.

Новостная рассылка Подкаст

Простые перестановки

Из всех перестановок от 1 до 9, используемых для получения девятизначных чисел, сколько главные?

Новостная рассылка Подкаст

4-значный код сейфа гостиничного номера

Можете ли вы определить, какой номер я использовал, чтобы запирать и отпирать сейф в номере отеля?

Новостная рассылка Подкаст

Цвет глаз хора

Выясните процент участников хора, у которых нет голубых глаз данные подсказки.

Новостная рассылка Подкаст

Клавиши калькулятора в углах Прямоугольник

Вопрос про четыре клавиши по углам прямоугольника на калькуляторе.

Новостная рассылка Подкаст

Среднее количество домов

Определите номера домов по подсказкам о среднем значении, медиане и моде.

Новостная рассылка Подкаст

Часы в зеркале

Какое время на самом деле показывали часы в зеркале?

Новостная рассылка Подкаст

Восхитительно делимый

Найдите панцифровое число, которое восхитительно делится.

Новостная рассылка Подкаст

Пять целых чисел с произведением 12

Сможете ли вы найти пять целых чисел, которые при умножении дают двенадцать?

Новостная рассылка Подкаст

Квадрат в прямоугольнике

Какой самый большой квадрат можно нарисовать в углу 10 см на 15 см? прямоугольник?

Новостная рассылка Подкаст

Лондонский марафон

A вопрос о средней скорости, необходимой для второй половины марафон.

Новостная рассылка Подкаст

Стрижка газона

Айнюк и Айли вырезали половину газона. Какова длина каждой стороны квадратный газон?

Новостная рассылка Подкаст

Найден недостающий фунт

Действительно замечательный ответ на загадку о пропавшем фунте.

Новостная рассылка Подкаст

Сорок пять в четырех частях

Разделите число 45 на четыре части в соответствии с предоставленной информацией.

Новостная рассылка Подкаст

Сморщенные палочки

Определите вес картофеля, оставленного на солнце. сушить.

Новостная рассылка Подкаст

Ошибка транспозиции

Определите банковский баланс с учетом информации об ошибке транспонирования.

Новостная рассылка Подкаст

Подсчет овец

Работа из числа имеющихся у Перси и Пэтси овец.

Новостная рассылка Подкаст

Жонглирование таймерами для яиц

Сможете ли вы рассчитать ровно девять минут, используя четырех- и семиминутное яйцо? таймеры?

Новостная рассылка Подкаст

Сумма перестановок

Какова сумма всех четырехзначных чисел, содержащих все цифры, единица до четырех?

Новостная рассылка Подкаст

Нечетная вероятность

Какова вероятность того, что два случайных числа будут одинаковыми, даже если они не оба ли странные?

Новостная рассылка Подкаст

Области в кругах

Подсчитайте количество областей в круге, образованном пересекающимися хордами.

Новостная рассылка Подкаст

Братья и сестры

Можете ли вы определить количество детей в семье Нумлов, учитывая подсказки о братьях и сестрах?

Новостная рассылка Подкаст

Среднее значение за экзамен

Что оценка требуется на последнем экзамене для достижения 80% общего среднего?

Новостная рассылка Подкаст

Торт нарезанный

Где следует ли разрезать последний кусок торта, чтобы получить два равных куска?

Новостная рассылка Подкаст

Священная сфера

Вычислите оставшийся объем сферы после того, как цилиндрическое отверстие пробурено через центр

Новостная рассылка Подкаст

Блоха в прыжках

Как много разных мест может найти блоха после 8-футового прыжка на север, юг, восток или запад?

Новостная рассылка Подкаст

Последняя цифра

Сколько положительные двузначные числа, квадрат и куб которых оканчиваются на такая же цифра?

Новостная рассылка Подкаст

Центральный вокзал

вероятность того, что следующий поезд отправится на север, составляет пять раз вероятность того, что следующий поезд отправится на юг.

Новостная рассылка Подкаст

Столовая Очередь

Это Можно ли ответить на вопрос, если возраст Бетси неизвестен?

Новостная рассылка Подкаст

Разлученные близнецы

Разработайте комбинацию сейфа по подсказкам о парах чисел.

Новостная рассылка Подкаст

делится на три

Головоломка о двухзначных числах, которые можно составить из десяти разных цифр.

Новостная рассылка Подкаст

Буквы в цифрах

Совершенно новая головоломка, в которой буквы в числах записываются как слова.

Новостная рассылка Подкаст

Квадратный треугольник

Углы треугольника — это квадратные числа. Кто они такие?

Новостная рассылка Подкаст

Головоломка с тремя перекрестками

Какова вероятность того, что три машины не дойдут до развязки? попасть в аварию?

Новостная рассылка Подкаст

Два основных квадрата

Какое наименьшее квадратное число (больше единицы) не может быть выражается как сумма двух простых чисел?

Новостная рассылка Подкаст

Недостающий фунт

Куда делся недостающий фунт в этой истории о трех людях, посетивших ресторан?

Новостная рассылка Подкаст

Клещи, тактики, галстуки и вытачки

Найдите в данной информации, как клещи соотносятся с тактиками, закрепками и складками.

Новостная рассылка Подкаст

Сила Рождества

Вопрос об индексах, который заставит вас задуматься математически на этом праздничном время года.

Новостная рассылка Подкаст

Муравей и дека

Что Единственный вопрос, который Дек мог бы задать Муравью, чтобы узнать, о чем он думает?

Новостная рассылка Подкаст

Книжный червь

Как далеко путешествует ли книжный червь во время еды?

Новостная рассылка Подкаст

Три математика

Как может третий математик быть настолько уверенным, что все хотят выпить?

Новостная рассылка Подкаст

Незаконченная игра

Если игра с подбрасыванием монет была прервана, как бы вы поделились выигрышем?

Новостная рассылка Подкаст

Буквенно-цифровой

Вопросы о буквах, используемых в натуральных числах.

Новостная рассылка Подкаст

Лучшие кости

Какие из необычные кости, которые вы выбрали бы, чтобы дать вам наилучшие шансы на выигрыш приз?

Новостная рассылка Подкаст

Проблема дня рождения

Какова вероятность того, что у двух или более учеников в классе будут одинаковые день рождения?

Новостная рассылка Подкаст

Гласные буквы Enigma

A Вопрос по кодированию навеян фильмом «Игра в имитацию».

Новостная рассылка Подкаст

Смещенная монета

A математический вопрос, который задают интервьюеры Microsoft и Google.

Новостная рассылка Подкаст

Двенадцать дней Рождества

Сможете ли вы точно подсчитать, сколько подарков посылает настоящая любовь во время двенадцать дней рождественских праздников?

Новостная рассылка Подкаст

За медведем

A головоломка о исследователе, которого преследует медведь, а также вопрос о английские и метрические меры с помощью Measurement Man

Новостная рассылка Подкаст

Подставки для Хэллоуина

A головоломка о том, почему Хэллоуин похож на Рождество вместе с новостями о новом Опись умений и навыков

Новостная рассылка Подкаст

Torch Tunnel

A головоломка про четырех человек, пробирающихся через туннель с одним факел вместе с новостями новой нумерологии страница

Новостная рассылка Подкаст

Кубик в молоке

A головоломка о кубике, который опускают в ведро с молоком, а также новости о новые маневровые пазлы

Новостная рассылка Подкаст


Следующие головоломки относятся к Transum Advanced Starters.

Алгебраическое произведение

Найти значение выражения проще, чем вы думаете!

Продвинутый стартер

Угловое мышление

Найдите диапазон возможных углов x, для которых tan x> cos x> sin x

Продвинутый стартер

Средняя скорость езды на велосипеде

Определите среднюю скорость двух поездок. Очевидный ответ не является правильным.

Продвинутый стартер

Вернуться на завод

Найдите все числа ниже 1000, которые имеют ровно 20 множителей

Продвинутый стартер

Barmy BIDMAS

Вводящий в заблуждение способ сформулировать ответ на простой расчет.

Продвинутый стартер

Парадокс коробки Бертрана

Парадокс коробки Бертрана — парадокс элементарной теории вероятностей, впервые сформулированный Джозефом Бертраном в 1889 году

Продвинутый стартер

Best Dice

Какие из необычных игральных костей вы бы выбрали, чтобы дать вам больше шансов выиграть приз?

Продвинутый стартер

Неравенства автомобилей

Решите три одновременных неравенства, чтобы определить, сколько автомобилей у меня есть.

Продвинутый стартер

Charging Rhinos

Найдите простой способ решить эту кинематическую задачу с участием мухи и двух носорогов.

Продвинутый стартер

День рождения Шерил

Используйте процесс исключения, чтобы определить правильную дату из предоставленных подсказок.

Продвинутый стартер

Координатное расстояние

Найдите k, учитывая, что (-2, k) находится на расстоянии 13 единиц от (10,9)

Продвинутый стартер

Кубоид

Найдите размеры кубоида, соответствующие приведенному описанию

Продвинутый стартер

Делится на 11

Можете ли вы доказать, что трехзначное число, первая и третья цифры которого в сумме дают значение второй цифры, должно делиться на одиннадцать?

Продвинутый стартер

Двойная или половина?

При изменении десяти процентов в день удвоение достигается быстрее, чем уменьшение вдвое?

Продвинутый стартер

Превышает на 99

Найдите число, удвоение которого превышает его половину ровно на 99.

Продвинутый стартер

Оптимизация забора

Найдите длину прямоугольника, охватывающего максимально возможную площадь.

Продвинутый стартер

Задача Ферми

Классическая задача Ферми с использованием стандартных методов оценки

Продвинутый стартер

Найдите радиус

Найдите радиус круга из небольшого количества предоставленной информации.

Продвинутый стартер

Деление на четыре дроби

Объясните, почему ответом на серию дробных делений является целое число.

Продвинутый стартер

GDC Challenge

Построить данный график на калькуляторе с графическим дисплеем

Продвинутый стартер

Geometry Snack

Найдите значение отмеченного угла на этой диаграмме из книги Geometry Snacks

Продвинутый стартер

Бабушка

Как далеко ушла бы бабушка после достаточно большого количества дней, учитывая ее режим ходьбы?

Продвинутый стартер

Руки вместе

Стрелки часов вместе в полночь.В какое время они вместе?

Продвинутый стартер

HCF и LCM даны

Если даны HCF, LCM и меньшее из двух чисел, можете ли вы найти другое?

Продвинутый стартер

Сколько левшей?

Определите количество членов, если дана вероятность случайного выбора левых элементов.

Продвинутый стартер

Сто пятьдесят процентов

Разделите 110 на две части так, чтобы большая часть составляла 150% меньшей части.

Продвинутый стартер

Key Eleven

Докажите, что построенное определенным образом четырехзначное число будет кратно одиннадцати.

Продвинутый стартер

Log Perfection

Определите, верны ли данные утверждения, содержащие логарифмы

Продвинутый стартер

Логарифмическое уравнение

Решите уравнение, содержащее логарифмы с разными основаниями

Продвинутый стартер

Максимум продукта

В сумме два числа дают 10.Какой самый крупный продукт, который они могли бы иметь?

Продвинутый стартер

Кратное сумме цифр

Какое число в шесть раз больше суммы его цифр?

Продвинутый стартер

Девятизначные числа

Сколько у них различных девятизначных чисел, которые содержат каждую из цифр от одного до девяти?

Продвинутый стартер

Пол другого ребенка

Какова вероятность того, что второй ребенок тоже мальчик?

Продвинутый стартер

Соотношение сторон

Рассчитайте соотношение сторон листа бумаги формата A4 без каких-либо измерений.

Продвинутый стартер

Paper Surprising Perimeter

Найдите периметр сложенного листа бумаги формата A4, как описано в этом коротком видео.

Продвинутый стартер

Параллельные графики

Определите по их уравнениям, какие из прямых графиков параллельны и перпендикулярны.

Продвинутый стартер

Мешки для пенни

Можете ли вы поместить 63 пенни в мешки таким образом, чтобы вы могли раздать любую сумму денег (от 1 пенни до 63 пенни), отдав выбор из этих расфасованных пакетов?

Продвинутый стартер

Perennial Rivals

Какая футбольная команда первой выиграет четыре матча?

Продвинутый стартер

Перестановочные функции

Найдите пары функций, которые коммутативны относительно композиции.

Продвинутый стартер

Отрезок веревки

Найдите место, где нужно разрезать кусок веревки так, чтобы получился круг и квадрат равных площадей.

Продвинутый стартер

Pizza Slice

Задача, которую можно решить, рассматривая площади треугольника и сектора круга.

Продвинутый стартер

Произведение индексов

Найдите произведение неизвестных индексов, входящих в два уравнения

Продвинутый стартер

Середины четырехугольника

Какая форма создается, когда середины сторон четырехугольника соединяются вместе?

Продвинутый стартер

Сдержанная блоха

Сколько разных мест может найти блоха после 8-футовых прыжков на север, юг, восток или запад?

Продвинутый стартер

Обратное соединение

Найдите общее правило для разницы между двузначным числом и тем же числом с перевернутыми цифрами.

Продвинутый стартер

Рис на шахматной доске

Сколько зерен риса находится на шахматной доске, если в каждом квадрате содержится вдвое больше зерен, чем в предыдущем квадрате.

Продвинутый стартер

Богатые или бедные?

Интересный результат процентного увеличения и уменьшения

Продвинутый стартер

Дорожные соединения

Спроектируйте дороги для соединения четырех домов, которые находятся на углах квадрата со стороной длиной в одну милю, чтобы минимизировать общую длину дорог.

Продвинутый стартер

Сумма одного и того же ряда

Найдите арифметический ряд и геометрический ряд, которые имеют одинаковую сумму первых пяти членов.

Продвинутый стартер

Одинаковые три цифры

Найдите выражения, содержащие индексы, и оцените числа, содержащие одинаковые цифры.

Продвинутый стартер

Семнадцать верблюдов

Объясните математику классической истории разделения дробей девятнадцатого века.

Продвинутый стартер

Single Fraction

Упростите выражение, включающее дроби, показатели степени и квадратный корень.

Продвинутый стартер

Круги скорости

Найдите диаметры окружностей в углах квадрата.

Продвинутый стартер

Отверстие в сфере

Найдите объем оставшейся части сферы после того, как в ней было просверлено 10-сантиметровое цилиндрическое отверстие.

Продвинутый стартер

Квадрат в прямоугольнике

Найдите площадь квадрата, нарисованного под диагональю прямоугольника

Продвинутый стартер

Tan 22,5

Найдите точное значение tan 22,50 без использования калькулятора.

Продвинутый стартер

Результаты тестов

Изучите заблуждение о том, что при сложении дробей вы складываете как числители, так и знаменатели

Продвинутый стартер

Три прямоугольных треугольника

Вычислите длины сторон без меток этих прямоугольных треугольников.

Продвинутый стартер

Transum Tonic

Какое наибольшее количество флаконов невозможно купить, если они поставляются в упаковках по 6, 9 и 20?

Продвинутый стартер

Tri-Junction

Реальная жизненная ситуация, которую можно проанализировать с помощью древовидной диаграммы.

Продвинутый стартер

Треугольник или четырехугольник

Может ли четырехугольник иметь прямой угол?

Продвинутый стартер

Два равно одному

Что не так с алгебраическими рассуждениями, которые показывают, что 2 = 1?

Продвинутый стартер

Два действительных числа

Сумма обратных двух действительных чисел равна -1, а сумма их кубиков равна 4.Кто они такие?

Продвинутый стартер

Незаконченная игра

Разделите приз в справедливом соотношении, в зависимости от вероятности желания каждого игрока.

Продвинутый стартер

Unlucky Seven Eleven

Следуйте инструкциям, чтобы умножить выбранное число, а затем объясните полученный результат.

Продвинутый стартер

Код гласных

Сколько способов вы можете создать код для гласных, назначив каждой гласной разные гласные?

Продвинутый стартер

Сводки погоды

Какие пять различных целых чисел умножаются и дают 12?

Продвинутый стартер

Какой вопрос?

Запишите все возможные вопросы, которые можно было бы задать, если бы это была диаграмма из учебника математики.

Продвинутый стартер

X Разделено на 2 года

Почему разные калькуляторы не согласовывают порядок операций?

Продвинутый стартер

Мои лучшие математические и логические головоломки (Math & Logic Puzzles) (Мягкая обложка)

5,95 долл. США

НЕТ В МАГАЗИНЕ, но мы, вероятно, сможем заказать его для вас

Описание


За 25 лет работы в качестве автора колонки «Математические игры» для журнала Scientific American Мартин Гарднер каждые шесть месяцев посвящал колонку коротким математическим задачам или головоломкам.Он был особенно внимателен к представлению новых и незнакомых головоломок, которые не вошли в такие классические сборники, как коллекции Сэма Лойда и Генри Дудени. Позже эти головоломки были опубликованы в сборниках книг, включающих отзывы читателей об альтернативных решениях или интересных обобщениях.
Настоящий том содержит богатый выбор из 70 лучших из этих головоломок, в некоторых случаях включая ссылки на новые разработки, связанные с головоломкой. Теперь энтузиасты могут бросить вызов своим навыкам решения и потревожить свое эго с помощью таких стимулирующих умов, как The Returning Explorer, The Matilated Chessboard, Scrambled Box Tops, The Fork in the Road, Bronx vs.Brooklyn, Touching Cigarettes и 64 других задачи, связанных с логикой и базовой математикой. Решения включены.

Об авторе


Мартин Гарднер был известным автором, опубликовавшим более 70 книг по предметам от естественных наук и математики до поэзии и религии. Он также всю жизнь увлекался фокусами и головоломками. Хорошо известный своей колонкой математических игр в Scientific American и своей «Уловкой месяца» в журнале Physics Teacher, Гарднер привлек лояльных последователей своим умом, остроумием и воображением.Мартин Гарднер: воспоминания Мировое математическое сообщество было опечалено смертью Мартина Гарднера 22 мая 2010 года. Мартину было 95 лет, когда он умер, и за свою долгую жизнь писателем он написал 70 или 80 книг. Первые книги Мартина в Дувре были опубликованы в 1956 и 1957 годах: «Математика, магия и тайна», одна из первых популярных книг об интеллектуальном волнении математики, которая достигла широкой аудитории, и «Причуды и заблуждения во имя науки», безусловно, одна из первых. популярные книги, чтобы бросить сокрушительно скептический взгляд на утверждения о лженауке и на множество обличий, в которых современный мир породил ее.Обе эти новаторские книги до сих пор издаются в Dover вместе с более чем дюжиной других названий книг Мартина. Они охватывают весь спектр от его элементарных кодов, шифров и секретного письма, которым пользовались поколения молодых читателей с 1980-х годов, до более требовательной книги «Новая амбидекстровая вселенная: симметрия и асимметрия от зеркальных отражений до суперструн», которую Довер опубликовал в своей книге. окончательная редакция формы — в 2005 году. Однако для тех из нас, кто был связан с Дувром в течение длительного времени, Мартин был больше, чем писателем, хотя и чрезвычайно популярным и успешным.Как член небольшой группы давних советников и консультантов, в которую входили Моррис Клайн из Нью-Йоркского университета по математике, И. Бернард Коэн из Гарварда по истории науки и Дж. П. Ден Хартог из Массачусетского технологического института по инженерии, советы и редакционные предложения Мартина 1950-е годы помогли определить издательскую программу Dover и дать ей точку зрения, которая — несмотря на многие изменения, новые направления и последствия эволюции — продолжает действовать и сегодня. По словам автора: «Ожидается, что политики, агенты по недвижимости, продавцы подержанных автомобилей и составители рекламных текстов будут распространять факты в корыстных целях, но ученые, которые фальсифицируют свои результаты, рассматриваются их коллегами как совершающие непростительные преступление.И все же печальный факт состоит в том, что история науки изобилует случаями откровенной подделки и случаями, когда ученые неосознанно искажали свою работу, рассматривая ее через призму своих страстных убеждений ».« Удивительная часть математиков — опытные музыканты. Это потому, что у музыки и математики есть прекрасные закономерности? »- Мартин Гарднер

Подробнее о продукте
ISBN: 9780486281520
ISBN-10: 0486281523
Издатель: Dover Publications
Дата публикации: 1 ноября 1994 г.
Страниц: 96 Английский язык
Математические и логические головоломки
Рекомендуемый уровень чтения
Минимальный возраст: 9
Максимальный возраст: 12
Минимальный уровень оценки: 4
Максимальный уровень оценки: 7
Категории
Связанные издания (все) .

Добавить комментарий

Ваш адрес email не будет опубликован. Обязательные поля помечены *